Women's Health Complete Objectives

¡Supera tus tareas y exámenes ahora con Quizwiz!

Labor Stage 3

(a) Uterus shrinks after delivery, less wall space for placenta to attach. It tries to thicken at first but then buckles under pressure and detaches (detachment often forms a large hematoma that follows the placenta) (b) Uterus should continue to contract and expel placenta (c) May need to have assistance with gentle traction on the umbilical cord (grossest tug-o-war ever) (d) Should always check for 3 vessel cord (e) Examine placenta for any abnormalities Photo: A is before delivery, B is shortly after

Differentiate between the types of urinary incontinence, include Tx (stress, urge, *overflow*).

*2 Types* - bladder outlet obstruction vs. atonic bladder *Bladder outlet obstruction* ■ pelvic tumor, stool impaction, anything large enough to obstruct the outflow ■ Post-Void Residual (PVR) is high; mechanism: normal bladder cannot overcome high outlet pressure (due to obstruction) ■ treat by removing the obstruction *Atonic bladder* ■ weak bladder cannot overcome normal outlet pressure, caused by diabetes, neuropathy, prolonged obstruction ■ treatment: clean intermittent cath (avoid indwelling catheters:infections big concern)

Describe the physiology of the menstrual cycle including the roles of the hypothalamus, pituitary gland, ovary and endometrium.

*2 cell hypothesis* 1. Interaction between the thecal and granulosa cells 2. Thecal cells make androgens in response to LH 3. Granulosa cells convert androgens to estrogens in response to FSH and aromatase. 4. After LH surge, follicle reorganizes to granulosa-thecal cells (luteinization) and forms the corpus luteum (CL) 5. CL is maintained by hCG binding to the LH/hCG receptor with production of progesterone. LH can only maintain the CL for a period of 2 weeks. (FYI LH/hCG is the ONLY receptor) *Hypothalamus* -Makes GnRH -> GnRH stimulates production of FSH and LH from pituitary gland -GnRH begins pulsatile secretion during puberty, prior to puberty is it released in a constant rate -GnRH is a very small decapeptide with a half life of only 1.5min. (is metabolized VERY fast!!) -Fast frequency assoc. with LH release, slow frequency with FSH release from pituitary -GnRH travels via the hypothalamic-pituitary portal tract to bind with pituitary GnRH receptors (Can't be released into circulation because it will be metabolized too fast!) -Estradiol plus progesterone slows down GnRH pulsatility *Pituitary Gland* -Based off of frequency of GnRH from hypothalamus, anterior pituitary will syn and release LH and FSH -> both act on ovary (slow frequency=FSH stockpile) -Estradiol and progesterone inhibit LH and FSH release- EXCEPT in the pre-ovulation period where the hypothalamus and pituitary, for a short period of time, have a pos feedback, to allow the LH surge *Ovary* Follicular Phase -FSH rises in late luteal phase (because corpus is going away), peaks on day 3 of cycle -FSH activates granulosa cells selected by the presence of FSH receptors -Dominant follicle (the chosen one!) is selected by changes in the microenvironment and neg feedback of estradiol and inhibin on FSH release. Loss of FSH causes atresia in follicles with reduced FSH receptors. -Estradiol production of 200pg/ml for 2 days triggers LH (Egg is released after LH surge) *Endometrium* -Endometrial characteristics are partitioned into the proliferative phase (preovulatory) and secretory phase (postovulatory) -Estradiol allows proliferation of the endometrium and stimulates progesterone receptors -Progesterone inhibits estradiol induced proliferation, causes the endometrium to have a secretory appearance -Withdrawal of estrogen and progesterone induce menstruation- constriction of spiral arteries, influx of PMNs and apoptosis

Discuss the epidemiology of PCOS.

*4-12%* of female population. Direct cause is not understood but there is a genetic factor involved Moms who have it likely have a daughter who will have it (25-50%) If you see an adolescent with hirsutism, treatment resistant acne, menstrual irregularities or obesity this should be on your DDX! carb intolerance, polycystic ovaries with a thickened capsule

Construct an initial workup plan for a patient with complaints of AUB.

*ALWAYS START WITH PREG CHECK* Then Check: -CBC (clotting issue) -TSH (both Hypo/Hyper) -PT/PTT (coag studies) -PRL (prolactinoma) -GC/CT (got that clap) -Pap Smear (cerv. cx) If Pt has severe/ acute AUB get a *TVUS* (transvaginal US) If Pt is > 45 YO, Obese, H/O tamoxifen get a *EMB* (endometrial biopsy) *Hysteroscopy is the gold standard if you are suspicious of cancer* (IE the tamoxifen use) (view the uterus and can take biopsy) If the patient is bleeding and need medical treatment *use IV estrogen until bleeding stops* If the AUB is chronic and not due to severe pathology that can be fixed then try using COC (BC pills) to reduce

Discuss *rH incompatibility* in terms of pathophysiology. Determine *algorithm for giving anti-D immunoglobulin*.

*Alloimmunization*=immune response to foreign antigens after exposure to genetically different cells or tissues. It's a natural event in pregnancy There is usually some mixing of the cells that occurs in pregnancy (but not much before delivery) RH- mom's blood recognizes RH+ fetus' blood as "foreign invader" & mounts attack by creating Abs against babies blood (anti-D, Kell, Duffy, Lewis) (These are different types of antigens) Biggest Abs created are the first 3 *"kell kills, duffy dies, lewis lives"* -but need many of these Abs angry about baby blood to cause issue (mutiny) so... workup: 1. check for Abs (IgG only) at 1st prenatal visit via *Coombs test*: antibody testing that checks for Abs (i.e. kell) attached to RBCs and looks for rH D aka neg. blood type- lacks D antigen -->D is much more common. -->its easy to become sensitized to this in pregnancy (i.e. 1st tri. procedures, bleeding, mild trauma, deliveries) so you could test dad but you can't guarantee paternity so don't risk it -->D's affect subsequent pregnancies more than 1st pregnancy tx: *RHOGAM*= anti-D immune globulin prophylaxis; it basically hides babies cells to moms cells so that mom doesn't attack them -when to give it? -->if mom is Rh neg and she has sxs (bleeding) give mini-dose in 1st tri. then give another dose in 2nd tri. -->test baby at delivery. If baby is not moms blood type, give at delivery (when some blood exchange occurs) to prevent mom from making Abs (done to prevent problems in subsequent baby* -if Ab screen comes back positive for anti-D (aka her cells already want to attack babies cells) there's an algorithm depending on titer (conc.): if high & persistent-big workup to eval. blood flow, MCA, amniotic fluid. may need to do fetal blood transfusion via umbilical artery sequelae: -*hydrops fetalis*: d/t Rh incompatibility--> fetus RBCs undergo hemolysis--> fetal anemia & fluid accumulation (ascites, effusions-pleural or pericardial). -->*almost always fatal* -->can also be d/t non-immune causes (90%) i.e. syphilis

Summarize the physical symptomatology that accompanies the physiologic changes listed in #2 above. (perimenopause, menopause, and post-menopause.) *abnormal uterine bleeding*

*Anovulation* (oocyte is not released during a menstrual cycle) is usually the cause of AUB. Remember that hormone levels fluctuate during MT and *estrogen is high and progesterone is low* (due to less inhibin = more estrogen while no ovulation= little to no progesterone) This is a set up for *endometrial hyperplasia and carcinoma* Risk of endometrial cancer 10% (up from 0.1% earlier) Pregnancy is still a consideration until age *55* If a woman has not had a period in years and suddenly starts bleeding, *Check for cancer*

Discuss the types of approach to the caesarean section (incisional).

*Approach* - *Incision type matters!* 1. Most important incision = *Uterine incision* 2. If the uterus is opened in the low-transverse (same as abdomen), the contractile portion is not affected 3. If the fetus is not able to be reached through this incision, it is extended a. This can be a U, a J, or the most common "T" incision b. ALL OF THESE ENTER THE CONTRACTILE portion of the uterus c. FUTURE TRIALS OF LABOR are NOT recommended 4. Additionally, the type of closure performed is important. SINGLE layer is bad news. *Abdominal (transverse is most common)* 1. This is also called the Pfannenstiel or Maylard incision 2. Excellent for healing unless morbid obesity, multiple prior incisions 3. "Bikini cut" *Vertical incision* 1. Begins 2-3 cm above superior margin of symphysis

Summarize the physical symptomatology that accompanies the physiologic changes listed in #2 above. (perimenopause, menopause, and post-menopause.) *Cardiovascular and bone changes*

*Cardiovascular changes* CVD is the *leading cause of death in women over 50* Younger women have higher HDL (protective) The idea that estrogen deficiency causes CVD rates to increase at faster rates and could be decreased with HRT has been studied, but not proven. What is known is that it is better for womens heart health if HRT is started earlier and they dont get off/on over and over again *Bone changes* Decreases in bone mineral density (see next card)

Identify the *potential risks* associated with mono-zygotic di-zygotic pregnancies in the *antenatal period.*

*Chorionicity is a strong determinant of perinatal outcome* - Dichorionic /Diamniotic twin pregnancies = most favorable outcome - Monochorionic/diamniotic pregnancies = highest rate of complications (d/t TTS) Antenatal Complications: - Spontaneous abortion - Congenital anomalies - Preterm delivery - Impaired growth of one twin due to unequal placental sharing (Discordant growth is > 20% difference in wt.) - Gestational diabetes - Pregnancy induced hypertension - Placental abnormalities (e.g. low near the cervix) - Vanishing Twin (reduction of one embryo during 1st trimester)

Discuss the clinical presentation of a *Bartholin's Gland Duct abscess.*

*Clinical Manifestation:* - Fever - Pain - Swelling - Erythema - Fluctuant Mass - Purulent drainage DDX: Bartholin's gland tumor - Firm mass - Age > 40

Discuss the *clinical presentation* of *candidal vaginitis.*

*Clinical Manifestations* - Pruritus - Erythema - Satellite pustules (Show up other places then the vagina, i.e. pannus) - White, curd-like discharge (Cottage Cheese) - NO ODOR

Discuss the *clinical presentation of bacterial vaginitis*

*Clinical Manifestations* - Thin, gray discharge - *Prominent* vaginal odor (Rotting fish) - Minimal inflammation - NO PRURITUS

Summarize the ASCCP recommendations for abnormal PAP smear results: OVERT CANCER

*Colposcopy!* If biopsies fail to confirm --> diagnostic excisional LEEP - loop electrical excisional procedure Cold Knife Cone (exactly what it sounds like)

Given a patient with use of operative delivery, summarize the most common complications.

*Complications:* i. There are several criteria that must be met before operative delivery can be used i. These pertain to station, position *Risks are:* vaginal lacerations, pelvic floor disorders for mom i. For fetus: ii. Extra cranial lesions (cephalohematoma is biggest) iii. Shoulder dystocia iv. Clavicular fracture v. Scalp lacerations vi. Facial nerve injury vii. Brachial plexus injury

Given a clinical scenario, correctly choose a contraceptive method from: hormonal, *barrier*, intrauterine, post-coital, or sterilization.

*Condoms*: • Size, friction, expiration, and sun exposure matters *Cervical cap* • Also used with spermicide • Fits tightly against cervix • Patient must be comfortable with placing • Insert before intercourse as well *Diaphragm* • Must be properly fitted in your office • Patient must be comfortable inserting at home • Should be inserted PRIOR to intercourse • Should be used in tandem with spermicide or jelly • Leave in place for 6 hours after intercourse • UTIs are sometimes an issue - toxic shock syndrome risk *Others:* *Dental dam* - like cellophane wrap, can make one from a cut up condom, used for oral sex protection *Sponge* - Insert into vagina up to 24 hours before sex. Blocks cervix and secretes spermicide. Hormone free. *Withdrawal* - Just don't.... don't do it. aka "pull out". *Family Planning* - See Natural Family Planning below *Withdrawal* - so bad I put it twice. Seriously, all of your teen patients will try this, and they honestly think it's the same as taking pills. *Lactation* - breastfeeding may reduce your chance of conceiving, but by no means is it reliable. *Spermicide* - cream, foam, gel, suppositories, anything with a chemical that will kill sperm. The sperminator terminator.

Review *recommended vaccinations* during pregnancy.

*DO NOT GIVE LIVE ATTENUATED VACCINATES TO PREGGOS* -*definitely give to every pregnant mom:* Tdap combo w/ pertussis (btwn wk 27-36) *EVERY* pregnancy--> immunity transferred to baby via breast milk Influenza *non-mist* during flu season (Oct-Mar) -*do NOT give* (accord. to book): measles, mumps, varicella, HPV, smallpox, contraindicated in pregnancy -*if high risk (exposure):* -HAV& HBV, meningococcus, pneumococcus:

Summarize the stages of labor.

*Definition:* uterine contractions that bring about demonstrable effacement and dilatation of the cervix 1. *Stage of cervical effacement and dilation* 2. *Stage of fetal expulsion* 3. *Stage of placental separation and expulsion*

Construct a diagnostic plan for a patient suspected of a thromboembolic event.

*Diagnosis of VTE* -Initial diagnosis via compression USN, then MRI -D Dimer not effective in pregnancy *Diagnosis of PE* -EKG: right axis deviation, T wave inversion in anterior leads -CXR: normal to atelectasis, infiltrate, effusion -Massive pulmonary collapse -Imaging: *CT pulmonary angiography is gold standard*

Discuss the *Diagnosis of candidal vaginitis*.

*Diagnosis* - *Normal vaginal pH (3.7 - 4.2)* - *KOH preparation* (slide under a microscope SEE PHOTO) shows *budding hyphae* -check for diabetes and HIV - Culture (Only used who are having recurrent candidal infections (More than 3 in 1 year))

NOT AN OBJECTIVE: Describe the *diagnosis and treatment of Trichomoniasis*

*Diagnosis* - Pap smear - Saline preparation = Trichomonads (Whipping clear footballs SEE PHOTO) - Culture (rarely indicated in clinical practice) *Treatment:* Oral Metronidazole - Single 2 gram Dose THIS JUST IN, HOT OFF THE PRESS You can dose 500 mg bid for 7 days but who really cares right? *Other Crap* - Educate that it IS an STI and increases risk of PID and infertility - Treat sexual partners - Use condoms until both parties are cured - Report to the Health Department

Given a clinical scenario, select the correct *diagnostic method* for a patient with *Bacterial vaginitis*

*Diagnosis* - Vaginal pH > 4.5 - Positive amine test = KOH Whiff test (Smells like rotting fish) - Positive saline microscopy (*Clue Cells* AKA Dirty Cells) . Clue cells are the bottom left picture, normal is the top left photo

Distension Vs Displacement (not an objective)

*Distension* cystocele or rectocele- fascial attachments still present, identifiable due to *Lack of rugae* on prolavaginal wall *displacement*- fascial attachments torn off. Vaginal rugae *visable*. Both can occur during labor.

Review the risks for each type of anesthesia and analgesia in delivery.

*Epidural Risks* i. Hypotension ii. Fever iii. Pruritus iv. Failure to work v. Transient deceleration vi. High Spinal Blockade vii. Spinal Headache *Spinal Headache* i. *COMMON OCCURRENCE* ii. Risk goes up with multiple sticks, obesity iii. Hallmarked by pain that *worsens with sitting up or standing and IS COMPLETELY RELIEVED BY LYING FLAT* iv. *NOTHING ELSE helps* v. Fixed by drawing *patient's blood* and inserting it into epidural site (done by anesthesia) - blood patch (Preeclampsia headaches don't go away when you lay down)

NOT AN OBJECTIVE: Describe the *etiology and clinical manifestations of Trichomoniasis*

*Etiology* - STD (STI) - Incubation period is 4 to 28 days - Highly contagious *Clinical Manifestations* - Frequency - Dysuria - Dyspareunia - Erythema - Pruritus - *Yellow-green, frothy discharge* - *Strawberry cervix* (Punctate cervical hemorrhages) - *Vaginal pH > 4.5*

What are the 3 types of Hypogonadism, what labs are high and low in each, and where is the defect?

*Hypogonadism causes Amenorrhea* 3 Types of Hypogonadism: *Hypergonadotrophic*: LH/FSH (HIGH) Estrogen (Low) Problem- Ovary (cause is often unknown) *Hypogonadotrophic*: LH/FSH (Low) Estrogen (Low) Problem- Hypothalamus/pituitary (Eating disorder, stress induced etc.) *Eugonadotrophic*: LH/FSH (normal) Estrogen (normal) Neither have cyclicity though Problem-Varied. (PCOS, Ovarian tumor, thyroid disease, Hyperprolactinemia caused by pituitary tumor)

Describe the *gravity and parity* of a patient given an accurate history.

*KNOW THESE* they will definitely be important on exam and in practice *-Gravida: state of pregnancy* *-Parity: state of delivery >20 wks of gestation; it is NOT increased in # for twins/triplets etc just as stillbirths do NOT lower this* 1. *nulligravida*- never been pregnant, not currently pregnant 2. *gravida*- current pregnant OR has been pregnant in the past IRRESPECTIVE of the outcome 3. *primigravida*- have been pregnant *once* (irrespective of outcome) 4. *multigravida*- been pregnant *more than once* (irrespective of outcome) 5. *nullipara*- never completed pregnancy beyond 20 wks (2nd trimester). 3 options to fit in here: never been pregnant so you obviously you've never had a pregnancy > 20 wks, abortion (either elective or spontaneous), or ectopic. 6. *primipara*- has delivered *only one* fetus beyond 20 wks IRRESPECTIVE of outcome (stillbirth or alive) 7. *multipara*- delivered *at least 2* fetuses > 20 wks think:* Florida Power And Light*= Full-term, Pre-term, Abortions (Spontaneous & Planned), Living Kids -*Full term= 37 + 0* ex: Para 2-1-1-3= 2 full term deliveries, 1 pre term, 1 abortion, 3 living kids

Describe relative contraindications for LARCs

*Levonorgestrel intrauterine system, LNG-IUS, (Mirena IUD) and Nexplanon Implant,* contraindictions: • Pregnancy (recommend you remove IUD if possible, increased risk abortion) • Uterine abnormality • Acute PID • Cancer • AUB (abnormal uterine bleeding) unknown etiology • Current IUD • High Risk for PID • PostPartum endometritis/infected AB in last 3 months • Acute infection (GC/CT) • Acute Liver Disease • Progestin sensitive breast Cancer *Copper IUD ParaGard,* contradictions: • Pregnancy* (which is why this is good for emergency contraception) • Uterine Abnormality • Acute PID/High Risk for PID • PostPartum endometritis w/in 3 months • Uterine CA • AUB unknown • Wilson's Disease (cause they already have too much cooper) • Cervicitis due to GC/CT • Allergy to copper (Photo shows copper insertion)

Describe normal pelvic anatomy ( 5 ligaments of the uterus ).

*Ligaments:* Cardinal (lateral) - condenses at the cervix with the uterosacral to form the peri-cervical ring, *an important element* of apical support Uterosacral - stretches and tears, but *still useful* for suspension Broad - provides no additional support Round - provides minimal additional support Utero-ovarian - provides no additional support There is also the *arcus tendineus fascia* (fascia, so not one of the ligaments)- Provides support for anterior and posterior vagina

Discuss laboratory and imaging findings of the molar pregnancy.

*Markedly elevated quantitative serum HCG* (would be in the 100,000's within 48 hours) *Snowstorm or grape-like appearance on U/S*

Given a patient with use of caesarean section, summarize the most common intraoperative complications.

*Maternal C-Section Risks* i. Infection ii. Hemorrhage iii. Thromboembolism iv. Bladder laceration/ureteral injury v. Bowel damage *Fetal C section Risks* i. Less than 1% of C-sections have these ii. Skin laceration iii. Cephalohematoma iv. Clavicular fracture v. Skull fracture vi. Facial nerve palsy vii. Brachial nerve damage viii. **Most injuries occurred after failed vaginal or failed operative delivery

Discuss the epidemiology of cervical cancer.

*Most common GYN cancer worldwide* - Highest death rates in black women of low SES *Almost all stem from HPV infections* - Almost always asymptomatic in early and mid stages WHY DOES IT HAPPEN? - Most women readily clear HPV - Persistent infection may cause dysplastic cervical lesions - Combination of environment, immunity, and genetics may lead to invasive cancer *Consideration of the possibility of HPV oncoproteins* Tumor can be: EXOPHYTIC - arising from ectocervix ENDOPHYTIC - arising from ENDOCERVIX INFILTRATIVE - develops ulcerative lesions on the cervix as it grows more inward *Infiltrative is the worst one because it grows backward into the uterus and is easily missed!*

Describe the *etiology* of *candidal vaginitis.*

*Most common pathogens:* - C. albicans - C. tropicalis - C. glabrata *Predisposing Factors:* - Antibiotic Therapy -oral sex - Corticosteroid Therapy - Diabetes - Oral contraceptives - Pregnancy - Immunodeficiency disorder - Use of Vaginal oils - Before or after her period - Elderly

Differentiate between the types of urinary incontinence, include Tx (stress, *urge*, overflow).

*Most common*, both men and women Definition: incontinence preceded by urge to urinate, residual urine is normal (no retention); the detrusor muscle is irritated and is moving too fast → urges "overactive bladder" Causes 1)Idiopathic - 2)Bladder irritation (infection, Caffiene, EtoH 3)CNS impairment Tx: Treat irritants, Behavioral changes Anticholenergics can help, but we avoid giving in the elderly Mirabegron- Beta-3 adrenergic agonist Relaxes the bladder to accomodate more urine.

Summarize the medical therapies available for correction of ovarian dysfunction.

*NOTE:* The textbook goes incredibly into detail for this objective and goes on for a shit-ton of pages... This is my best attempt to scale things down. But if you feel like you just need to know more to understand things, then might I point you to Ch. 20 of the textbook. *ABBREVIATED VERSION of problems and fixes* a. *Hypothalamic:* GnRH or FSH b. *Chronic anovulation:* - Clomiphene citrate - cause hyperstimulation ovulation - Letrozole (aromatase inhibitor. also use for PCO and infertility! ) - newer drug replacing CC - Low dose gonadotropins - do IVF then! - Ovarian drilling only if they don't want IVF - Metformin - increase ovulation and preg rates w/ PCOS c. *Hyperprolactinemia* - dopamine agonist (Bromocriptine) d. *Surgery* - Lysis of adhesions in tubal disease e. *Assisted reproductive technologies (ART)*: IUI, TESE, ICSI, IVF *Tubal disorders* (see below #6) *PCOS infertility treatment* (see next card) *EXTENDED VERSION* 1. *Hyperprolactinemia* i. Cause: physiologic, pharmacologic, or secondary cause ii. TRX: Dopamine agonist, or surgery if resistant adenoma iii. If Pregnant: Only give dopamine agonist therapy if hyperprolactinemia is due to a pituitary lesion >10mm 2. *Hypothyroidism:* i. Females: oligomenorrhea and amenorrhea ii. TRX: Thyroxine iii. If Pregnant: No treatment recommended 3. *Ovulation induction:* i. Clomiphene Citrate (like Tamoxifen, but both an estrogen agonist and antagonist, each one acting when needed to work on axis to ultimately produce FSH). ii. Can turn non ovulators to normal ovulators, or normal ovulators into hyperovulators. (IVF) iii. Rx orally given 3-5 days after period starts (spontaneous or progestin induced) iv. If period was spontaneous, get hCG test because unclear if CC is teratogenic v. Dose is weight dependent, titrated up to 100mcg. If no results in 3-6 months, move on to next option. vi. *New drug great for PCOS ovulation induction: Letrozole* 4. *Insulin resistance:* i. Esp w/ PCOS women, theory that Metformin can help ovulation , cycle, and response to CC 5. *Gonadatropins:* i. When CC fails, next step is injections of exogenous gonadotropins ii. Goal is to normalize ovarian function iii. Correct dose varies and requires close monitoring (watch for hyperstimulaiton syndrome = painful swollen ovaries) iv. Made from FSH, LH, and hCG of postmenopausal women's urine?! v. Good candidates inclues: Hypothalmic/Pituitary anovulation, CC failure, IVF candidates, superovulation (this will help produce >1 egg per cycle) 6. *Tubal Factors:* i. *Distal Disease: TRX: neosalpingostomy (new hole in fallopian tube)* ii. *Proximal disease: TRX: fluoroscopic tubal cannulation* iii. *Adhesions: TRX: Adhesiolysis* iv. *TRX: IVF: depends on age. If distal disease present then need salpingectomy (remove fallopian tubes)* 7. *Anatomic problem* i. Appropriate surgery to fix site

Develop a management plan for a shoulder dystocia.

*PRIMARY GOAL IS TO REDUCE THE HEAD TO BODY DELIVERY TIME* - Remember that the cord is compressed during this time! *McRoberts Maneuver* (see photo) 1. Remove legs from stirrups, sharply flex onto abdomen 2. Suprapubic pressure at same time 3. Downward traction to fetal head *Deliberate fracture of anterior clavicle* 1. Difficult to perform but heals great and better than nerve injury/death 2. Perform after failed McRoberts

Discuss the options for anesthesia and analgesia in delivery.

*Parenteral Agents* i. Meperidine and Promethazine via IM route ii. Stadol IM iii. Fentanyl- Risk is newborn respiratory depression *Nitrous Oxide* i. RARELY used *Anesthesia - Nerve Blocks:* -*Pudendal Blocks* (see photo) 1. Relatively safe 2. Doesn't allow for extensive manipulation (operative deliveries) - *Paracervical Blocks* 1. Usually in 1st stage labor 2. Can cause fetal brady 3. Pudendal Nerve not blocked with this so additional analgesia required *Contraindications to Spinal Anesthesia* i. Maternal hypotension ii. Coagulopathy iii. Thrombocytopenia (usually varies a bit by hospital) iv. Heparin w/in 12 hours v. Sepsis without treatment vi. Spinal infection/cellulitis vii. Brain tumor causing intracranial pressure *Meds for Epidural or Spinal Block* i. *Epidural is constant catheder, not just 1 shot:* lidocaine, bupivacaine in hyperbaric solution (addition of glucose) ii. *Spinal block is just 1 injection:* increased dose of above plus addition of fentanyl to increase rate of onset and decrease shivering Benefits to spinal block: short procedure time, rapid blockade onset, and high success rate

Given a patient in labor, explain the indications for caesarean section.

*Prior C-section or pelvic surgery affecting uterine tissue* - Ex: full thickness myomectomy - Pelvic deformity *Dystocia Risk* - Cephalopelvic disproportion - Failed operative delivery *Fetal Jeopardy* - Nonreassuring tracing *Abnormal fetal presentation* - Including breech

Summarize the initial physical examination needs for a female patient with infertility.

*Questions to ask:* i. Regular periods? Time and quantity ii. History of STDs? iii. Relevant medical history? iv. Relevant associated symptoms (galactorrhea, hirsuitism) v. Smoke, drink, caffeine, drugs, etc...? vi. Family and personal conception history vii. Sexual and pubertal history viii. Have you kept a menstrual diary?? *Physical Examination* i. VS: especially height or weight out of normal range (ex Turner syndrome) ii. Look at the patient for cues to secondary diagnosis - Hirsutism, alopecia, acne = high androgen levels - Acanthosis nigricans = insulin resistance = PCOS - Galactorrhea = hyperprolactinemia - Signs of thyroid abnormalities iii. Pelvic examination - Can't place Speculum? Infrequent sex, vaginismus - Look of vagina? Should be moist and rugated (= good estrogen lvl) - Cervix? Pink, normal amount of mucus (= good estrogen lvl) - Uterus shape? Irregular shape maybe leiomyoma, fixed = scarring= endometriosis or prior PID - Masses? Malignancy? iv. Cervical cancer screening up to date? v. Culture for gonorrhoeae/chlamydia vi. PAP smear vii. Breast exam? Is mammogram indicated by age or FHx?

Summarize the physical symptomatology that accompanies the physiologic changes listed in #2 above. (perimenopause, menopause, and post-menopause.) *Lower reproductive tract and sexual activity*

*Sexual activity changes* Can increase or drop-off Biggest factors are *libido and dyspareunia* Orgasmic dysfunction is also an issue but is frequently related to the above Psychosocial factors are important Partner involvement critical *Lower Reproductive Tract Changes* Vulvovaginal Changes: ATROPHY: dryness, itching, irritation, adhesions, shrinkage of the intrioitus Urogenital Changes: Prolapse Thinning All leads to dysuria, recurrent UTIs, sexual dysfunction

Short and Long term complications of PCOS (not an objective)

*Short-term consequences* Obesity Infertility Depression Sleep apnea Irregular menses Abnormal lipid levels Non-alcoholic fatty liver disease Hirsutism/acne/androgenic alopecia Insulin resistance acanthosis nigricans *Long-term consequences* Diabetes mellitus Endometrial cancer Cardiovascular disease

Discuss the clinical presentation of a thromboembolic event.

*Signs and Symptoms of VTE* -Almost always left sided due to compression of left iliac vein by right iliac and ovarian arteries -Abrupt onset of pain and edema -May have Homan's sign (pain in calf with dorsiflexion of foot) *Signs and Symptoms of PE* -Dyspnea, pleuritic chest pain, dyspnea with exertion, coughing -Tachypnea, apprehension (suspicion/fear of future), tachycardia

Summarize the physical symptomatology that accompanies the physiologic changes listed in #2 above. (perimenopause, menopause, and post-menopause.) *Skin Changes* and *Dental changes*

*Skin Changes* Thinning (can also lead to itching) Hyperpigmentation Wrinkles Loss of elasticity Decreased blood supply Diminished sebaceous gland secretion "(angry vagina)" *Dental changes* Buccal epithelium atrophies due to estrogen deprivation Decreased saliva Bad taste in the mouth Increased cavities Tooth loss Oral aveolar bone loss

Summarize the physical symptomatology that accompanies the physiologic changes listed in #2 above. (perimenopause, menopause, and post-menopause.) *Sleep changes and psychosocial changes*

*Sleep changes* Night Sweats (hot flashes) Light sleep Insomnia Sleep Disordered Breathing Many sleep disruptions are due to *other physiologic complaints*: Urinary complaints Heartburn Carpal tunnel *Psychosocial Changes* Risk of *depression* 2.5x higher than in premenopause) (finally something about psych) Due to high and erratic estradiol levels and Cultural Issues or feelings of an "End of an era" or a "Time of Loss". (though lots of ladies don't care)

Labor Stage 2

*Stage 2 you really try to access physical situation of the baby, aka, station* 1. Where is the fetal head? How can you *confirm lie* (fetal axis to mother axis)? Want it to be parallel *Physical exam* i. Perform vaginal exam ii. Do you feel sutures or buttocks when you touch through cervix (aka is baby breech?)? iii. Cephalic Presentations (picture) - Labor is normally A, B and C can lead to dystocia. "A" has smallest circumference through the canal, so it's ideal. *Imaging* i. Bedside USN c. Knowing the position of the fetus is a MUST before developing your delivery plan *Station* 1. Think of a line between the ischial spines of the mom-this is zero station 2. We go up to -5 station and down to +5 station - More common to say -4 and +3 - May also hear "floating" if the head doesn't engage at all - Prolonged floating is an indicator for a C/Section *Pushing* - Once fully dilated, most important force is mom's intraabdominal pressure. "pushing". - Pushing in first stage is useless and exhausts mom and could harm baby. - You should always wait till stage 2 to push. - Woman typically begin to feel when to bear down, contractions now last 1 minute and are 1.5 minutes apart. You push during contractions. - When Stage 2 is too long, it is a threat to mother and baby (There is a massive section in the text at this point, several chapters long about exactly what happens and what to consider towards the end of pushing and successfully delivering a baby, it could be it's own section, and I don't think we are expected to know the details) *Pop goes the weasel* Stage 2 is completed with fetal delivery

Discuss treatment options for a patient with ovarian cancer.

*Stage I-II disease* -Surgical resection is preferred (TVAH-BSO) -2 Chemo drugs can be attempted if they do not want to become permanently infertile (Carboplatin & Paclitaxel) *Stage III-IV disease* - Surgical cytoreduction to remove all gross disease Then -six courses of platinum-based chemotherapy Epithelial cancers are the most common type of ovarian. 5 year survival rate is 45% :( (OJC) Interestingly, BRCA mutation carriers have a better prognosis, due to increased platinum sensitivity (OJC) COC reduces risk of ovarian cancer

Review screening for each of the major STIs: *Syphilis*.

*Syphilis*- bad for fetus -commonly crosses placenta but can also get through lesions at delivery -sequelae: preterm labor, intrauterine fetal demise (IUFD), intrauterine growth restriction (IUGR), -death from anemia, thrombocytopenia, ascites, hydrous, myocarditis, nephrosis screening: -required by law at first prenatal visit. -high risk: should test again in 3rd trimester and delivery -lots of false positives tx: -2 doses of IM Pen G even if PCN allergy (exception= anaphylaxis tx w/ macrolides) -delivery: any lesion suspicious for syphilis in 3rd tri--> C/S 100% (do NOT wait for cx. they need C/S)

Describe the types of breech presentations and develop a management plan for each.

*The presenting part* is the portion of the fetal body that is in, or closest to, the cervical opening. The baby is either coming *head out (cephalic)* or *butt out (breech)* or *shoulder out* (baby axis is transverse to mom) *Frank breech* 1. Feet close to the head 2. Absolute buttock presentation LOL these kids keep trying to fold in half after being born. Stupid babies. *Complete breech* 1. One or both knees are flexed *Incomplete breech* 1. One or both knees are not flexed and one foot or knee is in the birth canal *Footling breech* 1. One or more feet below the breech *Management:* i. Can be delivered spontaneously, with partial operator help. ii. Can also attempt to turn fetus before labor iii. Generally use of maneuvers, traction, forceps to aid in delivery iv. *DONE IF YOU KNOW WHAT YOU ARE DOING* v. *IT TAKES A LONG TIME TO LEARN THESE*

Identify the normal stages of pubertal development.

*Thelarche* = Breast Development Mean age: 8-9y *Adrenarche/Pubarche* = Axillary/Pubic Hair Mean age: 9-10y (Adenarche can sometimes precede thelarche, most common in AA women) *Growth Spurt* Change in height after menarche = 3in. (Once a women menstruates she has attained given height- won't grow over 3in) *Menarche* Mean age: 12-13y Factors that determine onset of menses: -Biological age, -Body weight, % composition of body fat, -Family hx, -Maturation of CNS (development of + feedback response to estradiol)

Not specific objective, but important and from lecture: Info on Thromboembolic Events VTE = Venous Thromboembolism PE= Pulmonary Embolism c/s= c/section

*Thromboembolic events* -Significant cause of *mortality* in post partum period -10% of pregnancy related death in the US in last 16 years due to VTE and PE Pregnancy increases each of the risks of *Virchow's Triad* (injury to vascular endothelium, hypercoagulability, alteration in normal blood flow) -Women who have had a c/s have even greater risk if not early ambulation -Underlying genetic disorders also play a large role -Pregnancy is the highest likelihood of developing a clot

Predisposing factors to prolapse

*Trauma* - *Vaginal childbirth*, forceps *Natural Evolution* - Erect posture causes increased stress on muscles, nerves and connective tissue (no longer have the abdominal muscles to hold it up) *Hereditary* (genetic) predisposition- connective tissue disorders (Marfan, Ehlers-Danlos) Race: White > Black > Asian *Neurologic Injury* - pregnancy, childbirth, constipation *Menopause* (hypoestrogenic state) *Raised intra-abdominal pressure* - (e.g.: obesity, cough, constipation, lifting, etc) *Iatrogenic* - surgical procedure

Develop a medical management plan for thromboembolic event.

*Treatment of VTE* -In pregnancy, we use heparin -In the post partum period, we use heparin and warfarin -Graded ambulation and fitted stockings *Treatment of PE* -Difficult, can consider vena cava filter -In post partum period, treat with full anticoagulation

Given a clinical scenario, select the appropriate *treatment* for a patient with *bacterial vaginitis*.

*Treatment* Best: Oral metronidazole - 500 mg twice daily for 7 days: Don't drink on this med! Clinda is the backup drug if they are allergic Okay: Topical metronidazole - once daily x 5 days at bedtime - More expensive - Less likely to prevent systemic complications of BV

Given a clinical scenario, select the appropriate *treatment* for a patient with *candidal vaginitis*.

*Treatment* Topical anti-fungal agents (1st line) - Miconazole (OTC) Name Brand: Monostat - Clotrimazole (OTC) - Terconazole (RX) OR Oral anti-fungal agents - Fluconazole PLUS - Hydrocortisone ointment to relieve itching *Preventive Measures* - Avoid bubble baths - Avoid douching - Avoid us of vaginal oils - Wear cotton (or breathable) undergarments - Careful hygiene (shower after working out) *Treatment of Recurrent Infections* - Topical or oral agent monthly after menses OR - Oral meds weekly of 12 weeks

Summarize the approach to *treatment/management of Bartholin's Gland Duct abscess*.

*Treatment/Management* - Drainage - Antibiotics --> Doxycyline (100 mg BID) plus metronidazole (500 mg BID) --> Amoxicillin plus clavulanic acid (875 mg BID) - Sitz Baths *Correction and Prevention of recurrance* - Ward Catheter placement (Leave it in for 4 to 6 weeks. Allows time the tissue to scare down, like an earring) - Marsupialization (sewing the site of the abscess open)

Review the tier rating system for contraception.

*Try to stick to tier 1 or 2*. Rarely will you go into 3rd or 4th (only for medical reasons) *TOP TIER METHOD:* "It'll work" • Low unintended pregnancy rates • little or no user motivation • Longest duration of action • Requires shortest number of follow up visits *Includes:* IUD (LARC -long acting reversible contraceptive), implant, all forms of sterilization *SECOND TIER METHOD:* "It'll Probably work" • Higher failure rate subject to user error • Subject to patient remembering to take, reinsert, re-apply, return for injection, etc *Includes:* combination pill, single pill, patch, ring, shot *THIRD TIER METHOD:* "Eh, it should work" • Barrier methods • Requires patients to use consistently and correctly, they gotta understand man • Some of these methods are also based on timing of intercourse • Requires some dexterity of part of patient *Includes:* condoms, cervical cap, diaphragm, and fertility awareness methods *FOURTH TIER METHOD:* "Lol, good luck" • Highest Failure Rates • Include all the spermicidal preparations on their own • Failure rates around 21-20 percent • Note: does not include the withdrawal method. That method does not even have a rating!! It's just that bad.

Interpret findings of a wet prep.

*Wet Prep* - Clue Cells (Dirty Cells) = Bacterial Vaginitis - Fungal buds and Hyphae = Candidal Vaginitis - Trichomonads (Whipping Clear Footballs) = Trichomoniasis

OCC - *What does Estrogen do to the body?* NOT AN OBJECTIVE. but.... you should probably understand this.

*What does Estrogen do to the body?* - The estrogenic hormones are uniquely responsible for the growth and development of female sexual characteristics and reproduction in both humans and animals. The term "estrogen" includes a group of chemically similar hormones: estrone, estradiol (the most abundant in women of reproductive age) and estriol. Overall, estrogen is produced in the ovaries, adrenal glands and fat tissues. More specifically, the estradiol and estrone forms are produced primarily in the ovaries in premenopausal women, while estriol is produced by the placenta during pregnancy. In women, estrogen circulates in the bloodstream and binds to estrogen receptors on cells in targeted tissues, affecting not only the breasts and uterus, but also the brain, bone, liver, heart and other tissues. Estrogen controls growth of the uterine lining during the first part of the menstrual cycle, causes changes in the breasts during adolescence and pregnancy and regulates various other metabolic processes, including bone growth and cholesterol levels. *Estrogen on Bone* Estrogen produced by the ovaries helps prevent bone loss and works together with calcium, vitamin D and other hormones and minerals to build bones. Osteoporosis occurs when bones become too weak and brittle to support normal activities. Your body constantly builds and remodels bone through a process called resorption and deposition. Up until around age 30, your body makes more new bone than it breaks down. But once estrogen levels start to decline, this process slows. Thus, after menopause your body breaks down more bone than it rebuilds. In the years immediately after menopause, women may lose as much as 20 percent of their bone mass. Although the rate of bone loss eventually levels off after menopause, keeping bone structures strong and healthy to prevent osteoporosis becomes more of a challenge. *Estrogen on the Vagina and Urinary Tract* When estrogen levels are low, as in menopause, the vagina can become drier and the vaginal walls thinner, making sex painful. Additionally, the lining of the urethra, the tube that brings urine from the bladder to the outside of the body, thins. A small number of women may experience an increase in urinary tract infections (UTIs) that can be improved with the use of vaginal estrogen therapy.

Acquiring a History on Initial Prenatal Presentation (not a specific objective)

*What to Screen for at least once a trimester:* -barriers to care -nutritional status -unstable housing -desire for pregnancy -intimate partner violence -hx depression/psychosis -stress -substance use: tobacco, EtOH, illicit drugs ^^screening shown to decr. rate of gestational DM, PPROM, vaginal bleeding *smoking or 2nd hand smoke:* -incr. risk: placenta previa & abruption, PPROM, preterm, low birth weight, SIDS, spont. abortion, decr. HR variability, fetal death, asthma, childhood obesity -causes fetal hypoxia, reduced uteroplacental blood flow, direct toxic effects -FETAL NICOTINE EXPOSURE IS GREATER THAN THAT OF THE MOTHER =( -quitting at any stage can improve outcomes -counsel via *5 A's of smoking cessation:* Ask about smoking every visit, Advise the risks of smoking, Asses their willingness to attempt cessation, Assist w/ cessation materials (hotlines), Arrange to track abstinence progress -if they choose nicotine patch, safer but still observe carefully *alcohol* -potent teratogen that causes fetal syndrome of growth restriction, facial abnormalities, & CNS dsfxn -most likely in women 35-44 y/o, white, college grads, employed *illicit drugs* -10% of fetuses are exposed to at least 1 drug -causes fetal-growth restriction, low birth weight, & drug withdrawal soon after birth =( -always ask about this as women often do not admit to it if they seek prenatal care *intimate partner violence* -more prevalent than any major medical condition detectable through routine prenatal screen (other than preeclampsia) -incr risk of preterm, fetal-growth restriction, & perinatal death -hotline available so offer it

Define *Ectopic pregnancy:* Risk factors Most common location Signs/symptoms Labs/ Tests Treatment options

*What:* Pregnancy that is located in an area other than the uterus *Risk Factors:* PID, Tubal surgery (any scarring in the fallopian tube) *Most common location*: Ampulla (95%) *S/S:* -Abdominal/ pelvic pain (1 sided or both) -hypotension -Irregular bleeding -Hx of no period for a while (cause you pregs) -may be in hemorrhagic shock (if ruptured) -Phrenic nerve irritation (blood irritates nerve and causes shoulder pain?) *Labs:* -hCG (should double every 48 hrs so if its lower you'd be suspicious) *Tests:* -*Ultra-sound* (RING OF FIRE sign is BAD this indicates that the ectopic is impending rupture) (need surgery) *Tx:* -If the Pt is stable, small ectopic, and will comply with follow up then use *Methotrexate* : will not dissolve large ectopic -If the Pt is unstable, RING OF FIRE SIGN on US, large ectopic or unlikely to follow up then choose *Surgical* (Linear salpingostomy, Segmental resection, Salpingectomy)

Construct a plan for *counseling high risk patients regarding possible complications* (advanced maternal age patients, adolescent patients).

*adolescent patients:* -incr. risk of anemia, preterm delivery preeclampsia, DS (trisomy 21) -incr risk of *short interval conception*= getting pregnant w/in 365 days of last delivery; assoc. w/ incr. risk of premies -less likely to receive prenatal care

OCC: Amniocentesis & CVS

*amniocentesis:* -MC done by wks 15-20 but can be done anytime -purpose: fetal karyotyping (ie DS) -AVOID THE PLACENTA -results ~1 wk -may be therapeutic (polyhydramnios) -low risk: 2% chance of fetal loss, can cause intrauterine infxn, leakage of amniotic fluid *CVS: Chorionic Villus Sampling:* -done btwn 10-13 wks (earlier than amnio) -purpose: karyotyping & more -results in 2 days (earlier than amnio) -GOES INTO THE PLACENTA so gives more info -HIGHER fetal loss rates than amnio -lower rates of infxn bc not going into amniotic fluid -requires more skill than amnio both amnio & CVS are done via US guidance

OCC: Physiological Changes That Occur With Pregnancy

*changes that occur during pregnancy* -vaginal mucosa appears dark-blue red & congested= Chadwick sign -incr. cervical softening as pregnancy advances (also caused by combined contraceptives) -uterine size grows in anteroposterior diameter which feels dough or elastic on bimanual exam -softer fundus & compressible interposed softened isthmus- Hegar sign (softening can make the cervix & uterine body feel like separate organs) -soft, blowing sound that's synchronous w/ the maternal pulse may be heard w/ stethoscope= uterine souffle (caused by passage of blood through the dilated uterine vessels) -sharp, whistling sound that is synchronous w/ the fetal pulse= tunic souffle (caused by rush of blood through umbilical arteries) -fetal heart tones may be heard -fetal movement (felt 2 weeks earlier in subsequent pregnancies compared to 1st pregnancy)

Prenatal Care Intro (not an objective)

*epidemiology:* ~50 million prenatal visits/yr. -average: 12 visits/pregnancy (high risk or just paranoid 1st pregnancy moms have up to 17+ visits/pregnancy) *-->routine appt schedule: q4 wks until 28 wks then q2 wks until 36 wks then q1wk if uncomplicated pregnancy (need at least 8 visits)* *effectivity of prenatal care* -5x decreased risk of maternal death -2x decreased risk of preterm births -decreased stillbirths -decreased neonatal death associated w/ placenta previa (cervical os covered by placenta; sx= painless bleeding esp in 3rd tri.) & fetal-growth restriction -decreased postterm pregnancy *complixns during prenatal care* #1 MC: Gestational HTN (after 20 weeks) #2 MC: DM *barriers to care* #1 MC: late identifixn of pregnancy -others: $, age *avg durn. of pregnancy from 1st day of LMP= 40 wks; but full term= 37 + 0* -divided into *3 trimesters:* each is 14 wks long *1st Tri= until the end of the 14th week* (14 +7) *2nd Tri= wks 15-end of 28th* (15+0 to 28+7) *3rd Tri= wks 29-end of 42nd (but avg is 40)* *clinically, use week of gestation completed; pt's often just go by trimester* ^^ trimesters are useful bc certain OB problems tend to cluster to a certain tri & for tx options. ex: most spont. abortions occur in 1st tri. women w/ hypertensive disorders d/t pregnancy are often dx in 3rd tri

How long should someone be on HRT?

*for the purpose of the test via Dr. Garbas* follow the FDA recommendation that you give the pt the *lowest effective dose, and stop giving it when they no longer need it to treat their vasomotor Sx*. In real life HRT gives protection to your heart and your bones when given early and continuously, so some drs might prescribe it against FDA recommendations.... but doing that makes me nervous and opens you up to lawsuits.

Discuss *recommended lifestyle changes* during pregnancy.

*nutrition*: *-amount of appropriate weight gain is based on pre pregnancy BMI* -maternal weight gain during pregnancy influences birth weight -decreased caloric intake (starvation) during pregnancy --> impaired brain development -pre pregnancy obesity: incr risk of gestational HTN, preeclampsia, gestational DM, C-sections -guidelines: underweight pt: 28-40 lb gain normal wt pt: 15-20 lbs (book says 25-35 lbs...) obese patient: 11-20 lb gain Morbidly obese: do not gain any weight --pregnancy requires an addl. 80,000 cals total, mostly during the last 20 wks. -->to meet this, *caloric increase of 100-300 cals/day is recommended during pregnancy but not equally throughout (0, 340, 450 respectively to 1st, 2nd, & 3rd trimesters)* -*no raw food or unpasteurized dairy * -*caffeine: <200 mg daily (12 oz cup) allowe*; but NO energy drinks*; no association w/ miscarriage or preterm birth -water is 1st line tx for contractions d/t dehydration *vitamins* -excessive supplement use can cause nutrient toxicities (>10,000 IU Vit A/day may be teratogenic; Accutane is made from Vit. A derivative hence teratogenic) -but a *prenatal a day= good* -*Folic Acid: LEVEL A Recommendation= 400 ug/d thorough periconceptional period to prevent neural-tube defects (structure that brain & cord develop from)* if had hx of open neural tube defect, take 4 mg daily -Low B12 can also cause neural tube defects *exercise* -no need to limit exercise but don't excessively fatigue or risking injury. - impt to have thorough exam before recommending exercise program -recommended: regular, moderate-intensity activity at least 30 mins daily *sex* -if healthy pregnant woman, continue on

Summarize the clinical presentation, diagnostic criteria, & mngmnt for *pre-eclampsia*.

*pre-eclampsia= a syndrome of HTN plus proteinuria or evidence of multi-organ failure* *sxs./poor prognosis indicators:* -headaches -visual changes/disturbance -RUQ pain -any above indicate worsening pre-eclampsia *dx:* any combo of the below criteria: -persistently 1+ proteinuria on dipstick -Protein:Cr ratio > 0.3 - 300+mg PRO/24 hours -evidence of multi-organ failure: -->thrombocytopenia --> renal or liver dsfxn --> pulm. edema --> CNS disturbance: hyperreflexia, clonus, light sensitivity/auras ^^if any of these are evident, must deliver baby soon. *word of caution, labs may lag sxs by up to 24 hours...* *tx:* -same as gestational HTN *(alpha blockers)* -delivery likelihood increases quickly but try to get BP under control prior to inducing labor (labetalol) -*delivery is the only cure* -must weight risk of pre-term vs. organ damage to mom -*glucocorticoids*: help baby lungs mature faster -*IV Mg Sulfate*- used *short-term* to prevent seizures and used if eclampsia develops to tx seizures; *it will NOT lower BP* --->IV MgS only used short term to keep mom from seizing long enough for babies lungs to mature so you can induce deliver 1-2d later. AE= sluggish, must be on bedrest. fetus respiratory distress at very high doses *sequelae:* 1. progression into *eclampsia= preeclampsia sx's PLUS generalized tonic- clonic SEIZURES* in mom --->eclampsia effects on mom: placental abruption, neuro deficits, pulmonary edema, MI, acute renal failure 2. HELLP Syndrome- next card

Discuss the major risk factors for *pre-term birth*.

*preterm birth:* previously based on fetus weight but now on wks of gestation completed -Anything before 36 and 6 or 7th days (recall: full-term= 37+0) -further classified into early & late preterm: *Early preterm:* 33+ 6th or 7th days or earlier *Late preterm:* wk 34 to 36+7 *RF:* 1. Spontaneous unexplained Pre Term Labor with INTACT MEMBRANES(amniotic sac) -d/t: uterine distention (polyhydramnios, fibroids), maternal-fetal stress, premature cervical changes (ie d/t prior cervical surgery/shavings), intrauterine infxn 2. Idiopathic PPROM (Preterm premature rupture of the membranes: AKA your water breaks!!!! ) 3. Maternal or fetal indications/complixns 4. Multiple gestation: least common cause polydramnios= excess amniotic fluids Fibroids or other intracavitary lesions occupying space act like multiple gestation "wimpy white boy": white males are notorious for having immature lungs so try as hard as possible to get these moms closer to term *sequelae* -*necrotizing enterocolitis*= biggest problem; parts of bowel become necrotic. 2nd leading COD in premies after RDS (respiratory distress syndrome) -brain injury/intracranial hemorrhage -lung distress d/t immaturity -cerebral palsy -retinopathy -hearing loss -dental problems -sepsis

E

-

Define the terms: -Primary amenorrhea -Secondary amenorrhea -Menorrhagia -Metrorrhagia -Hypomenorrhea -Oligomenorrhea -Withdrawal bleeding Not an objective

- *Primary Amenorrhea* = No period by age 14 in the absence of growth spurt or secondary sexual characteristics. OR No period by age 16 in the presence of normal growth spurt and secondary sexual characteristics - *Secondary Amenorrhea* = You have had a period before, but now it has stopped. - *Menorrhagia* NOW termed heavy menstrual bleeding *(HMB)* = bleeding over 7 days or 80 ml - *Metrorrhagia* Now termed *intermenstrual bleeding* = bleeding at irregular intervals (May be termed breakthrough bleeding if associated with hormone usage) - *Hypomenorrhea* = Shortened duration of blood flow - *Oligomenorrhea* = Cycle interval longer than 35 days (little bleeding) - *Withdrawal Bleeding* = Occurs from abrupt decline in Progesterone levels

Compose a long term management plan for a patient with a molar pregnancy.

- *Suction evacuation* - Monitoring of serum HCG - Treat and evaluate for metastases - Chemotherapy meds for persistent GTD 1. Methotrexate (attacks rapidly dividing cells) 2. Actinomycin D - Effective contraception x 6 months - Recurrence risk - 10% - Early ultrasound in next pregnancy

OCC? I'm not sure - MEC Eligibility - PostPartum

- 20% of breast feeding moms will ovulate by 3 months post partum (PP) - If intermittent breast feeding, treat as if not breast feeding - If exclusively breast feeding, avoid estrogen agents until 6 weeks PP - IUD or POP/DMPA both have category 1 or 2 ratings -COC have been limited during the first 6 weeks due to risk VTE/DVT

Describe the clinical presentation of a patient with cervical cancer.

- Clinically, the first symptom of cervical cancer is *abnormal vaginal bleeding, usually postcoital.* - Vaginal discomfort, malodorous discharge, and dysuria are not uncommon. - If bladder and rectum are directly involved, it can present with constipation, hematuria, fistula, and ureteral obstruction. - Common sites for metastasis include *extrapelvic lymph nodes, liver, lung, and bone.*

Discuss the use of the HPV vaccines for the prevention of cervical cancer.

- Create local and humoral immunity - Create type specific humoral antibody production - Block new HPV infection DO NOT prevent transient HPV infection DO NOT resolve current HPV infection Fun Facts: Show nearly 100% efficacy in preventing HGSIL from the types of HPV in the vaccines HPV infections have dropped 56% since start

PCOS infertility

- Medical therapy (see below) - Weight loss - Clomiphene citrate starting at 50mg day 5-9 - Letrozole 2.5 -7.5mg day 5-9, like CC- 1st Choice - Metformin if needed - Human gonadotropins

OKC (only khayla cares)

- The uterine corpus is predominantly smooth muscle, whereas the cervix is primarily connective tissue. Cellular components of the cervix include fibroblasts, epithelia, and few smooth muscle cells. - They aren't exactly sure what is causing the pain of contractions. Tope 4 theories are; (1) hypoxia of the contracted myometrium—such as that with angina pectoris; (2) compression of nerve ganglia in the cervix and lower uterus by contracted interlocking muscle bundles; (3) cervical stretching during dilatation; and (4) stretching of the peritoneum overlying the fundus. - In the first stage of labor, the membranes, when intact, and the fetal presenting part serve to dilate the upper vagina. The most marked change consists of stretching of the levator ani muscle fibers. This is accompanied by thinning of the central portion of the perineum, which becomes transformed from a wedge-shaped, 5-cm-thick tissue mass to a thin, almost transparent membranous structure less than 1 cm thick. When the perineum is distended maximally, the anus becomes markedly dilated and presents an opening that varies from 2 to 3 cm in diameter and through which the anterior wall of the rectum bulges. THIS IS WHY YOU POOP AND RIP. - When a baby is born as the head is passing through the pelvic outlet, they name the stations by which direction the occipital is facing, both Left and Right, and with Posterior, Anterior and Transverse. Left Occiput Posterior is the most common devilvery style. Or LOP. - Ugly lump on top of the baby's head is first started from the portion of the scalp poking through the cervical opening during dilation. It becomes edematous due to pressure and is worsened as it passes through the canal. It is called caput succedaneum - In general there seems to be to "Right way" for a woman to go through labor. Standing, walking, laying in bed have no different outcomes. Being coached or silent room, no different outcomes. The only study with a difference was more adverse/negative outcomes for women who gave birth outside of a hospital because of unavailable appropriate medical care.

Describe precocious puberty and delayed puberty. -Not thoroughly covered in lecture, but I found in chapter 14 of book.

-*Precocious puberty* AKA *premature pubertal development*, has historically been defined as breast or pubic hair development in those *younger than 8 years*. However, some researchers noted that girls in the US overall are undergoing normal pubertal development at younger ages than previously reported. In addition, racial differences exist -> beginning earliest in black girls followed by Hispanic and white girls. Risks: This can stunt your growth, increase risk for cancer bc estrogen is in the body longer. -*Central precocious* (gonadotropin dependent), we see *malfunction in GNrH cycle*. Early activation of the hypothalamic-pituitary-ovarian axis leads to pulsatile GnRH secretion, increased gonadotropin formation, and in turn, increased gonadal steroid levels. RARE. Example: 6 year old with breast development and pubic hair. *give meds that stop the axis -*Peripheral precocious* (gonadotropin independent), we see development of androgenic effects due to *excess sex hormones*. We may also see germ cell tumor production (HcG in a non pregnant state) -Elevated estrogen levels may originate from a *peripheral source*, such as an ovarian cyst. Termed peripheral precocious puberty, this category is characterized by *lack of GnRH pulsatile release, low levels of pituitary gonadotropins, yet increased serum estrogen concentrations*.(MC cause- granulosa cell tumor) -Heterosexual precocious (inc androgen secretion), and temporal variations. -*Delayed Puberty* Puberty is considered delayed if *no secondary sexual characteristics are noted by age 13*, which is more than 2 standard deviations away from mean age, or if *menses have not commenced by age 16*. -Affects 3% adolescence -Constitutional delay is most common cause (probable cause is delay in reactivation of the GnRH pulse generator) -Anatomical defect (congenital or aquired)

What is a normal cycle?

-28 Days +/- 7 days -Period Lasts 5-7 days -Should not affect your daily life. AUB is anything outside of normal or more than 80 mL in a 7 day cycle

Discuss the unintended pregnancy rates in the U.S.

-45% of all U.S. pregnancies were unintentional • ¾ teen pregnancies.... see guys, abstinence only education works! -45 unintended pregnancies for every 1,000 women aged 15-44 • This is higher than any other developed country! -If a family is below the poverty line: • 112 per 1,000

Describe the clinical presentation of mastitis.

-Almost always unilateral -Usually appears in 3rd or 4th week post partum but can appear earlier -See with chills, rigor, fever, tachycardia -Breast will *hurt* and become *red, hard* -Some will go on to develop an abscess From book: Detection of fluctuation may be difficult, and sonography is usually diagnostic

Summarize *ultrasound timelines for a routine pregnancy*

-An indication for a 1st trimester US is to estimate gestational age via the *Crown-Rump Length (CRL): most accurate predictor of fetal age* (w/in 3-5 day error range) via measuring head to butt of fetus -in *1st tri*, if the US CRL and the LMP are w/in 1 week of each other, the LMP wins. *If they differ by > 1 wk, US CRL wins* -in later trimesters, the CRL range of difference increases ex: If LMP says due on March 15th & US CRL says due on Mar 31st, then US CRL wins. -other indicn. for 1st trimester US: confirm intrauterine pregnancy & nuchal lucency -->nuchal lucency: marker of DS in which fetus may have cardiac abnmlities -2nd tri US: eval. anatomy, growth, well-being (placenta sufficiency) --> often called the "tell me the sex US" --> often is a level 2 US, meaning its a 3D US --> *for high risk pregnancies, this 2nd trimester US MUST BE A 3D US* -late 3rd tri US: -Assess baby movement, fluid, see if baby is trying to breathe Biophysical profiles: gives a "score" to fetal well- being; Score: 2, 4, 6, 8, 10 *<6= scary*

OCC: other associated pregnancy conditions mentioned in the book

-Back pain--> tylenol PRN is fine -venous leg varicosities: may show signs of blemishes & mild discomfort at end of day that requires rest w/ feet elevation. tx= supportive: leg elevation, rest, stockings -hemorrhoids= rectal vein varicosities d/t pelvic peons pressure increase. tx- topical anesthetics, warm soaks, stool softeners -heartburn- very common in pregnancy d/t upward displacement & compression of stomach by uterus. tx- smaller more frequent meals, antacids. -pica: crave strange foods, maybe caused by severe iron deficiency -ptyalism (profuse salivation)- unexplained, often occurs after eating starch -fatigue -increased sleep needs vs. 3rd tri more awakenings & less REM sleep. tx: Benadryl & naps -leukorrhea- increased vaginal dschrg & incr mucous secretion by cervical glands d/t high estrogen -"iron, iodine, & folic acid* are the 3 vitamins diet alone may not be sufficient (iron supplementation not needed in 1st tri); iodine (from salt & bread) offsets the increased fetal requirements & maternal renal losses of iodine. iodine deficiency--> severe neuro deficits -27 mg iron supplement daily

OCC

-Between 6mo after birth and puberty reproduction is "shut off". -Remember GnRH is special.. small 10 amino acid structure that gets metabolized very fast. -Pool of 2-20 eggs are chosen from for ovulation, but only 1 gets competitive advantage and is released -# of eggs peaks when you are 8 weeks old as a fetus (8-10 million eggs). By birth there are 1 million and by puberty there are 300,000. -hCG is a "master key" -Luteinization, weird word, means transformation of ovarian follicle into corpus luteum after ovulation -Early pubertal development may be seen in both sexes, but females are much are commonly affected (23:1 ratio) -GnRH = Gonadotropin-releasing hormone -LH = Luteinizing hormone -FSH = Follicle-stimulating hormone -hCG = human chorionic gonadotropin (hCG is is similar structurally to LH, shares the same receptor as LH, and assumes the role of corpus luteum support during early pregnancy) -Dehydroepiandrosterone (DHEA), also known as androstenolone, is an endogenous steroid hormone. It is the most abundant circulating steroid hormone in humans.

Identify a patient with diastasis recti.

-Common occurrence of following pregnancy -Marked separation of the rectus abdominus muscles -Due to atony of the abdominal wall musculature -Exercise decreases the risk of this. (may also improve it post partum) -Resolution: may be spontaneous. Surgery is indicated if pt has significant symptoms

Develop an initial workup for a patient with chronic pelvic pain.

-H&P -Watch gait look at mobility -Point to pain -*Carnett's sign* pain when contracting abdominal muscles lifting head and shoulders (sign of ant. Abd wall pathology) -Pelvic exam(bimanual first) -Urinalysis, TSH, STI panels -If bowel symptoms --> work up bowel -If urinary symptoms --> consider cystitis (and do cystoscopy) . -neuro/psych -If you aren't finding anything then consider Laparoscopy -*Laparoscopy is the gold standard for chronic pelvic pain*

Diagnosis of ectopic

-Imaging via transvaginal -Ultrasound -Can see a gestational sac at 4.5-5 week -Yolk sac at 5-6 weeks -Cardiac activity at 5.5-6 weeks -Beware of corpus luteal cyst masquerading as adnexal mass or a pseudo gestational sac -Concerning: lack of IUP, adnexal mass, blood in cul-de-sac -Serology -Beta-HCG should rise (usually double) every 48 hours and continue until around a level of 100,000-if not rising by 72 hours concerning for ectopic

Lobular carcinoma in situ: not sure we need to know this

-Lobular cell proliferation and increased acini distention -Rarely diagnosed on exam or imaging -Increases the risk for developing overt breast cancer (usually in ipsilateral breast) -~1% per year (risk factors may increase) -Treat with surgical excision if diagnosed on needle biopsy -VERY ESTROGEN RECEPTOR POSITIVE

Describe the clinical presentation of a patient with PCOS.

-Often present a few years after puberty 1. Signs of hyperandrogenism (hirsutism, acne, elevated Test. level) 2. Multiple ovarian cysts on US 3. Menstrual irregularities or infertility Signs: -Obesity -Hairy -signs or levels of testosterone too high (acne) -Amenorrhea -Unpredictable and heavy menstrual bleeding

Placental Adherence Syndromes (a cause of PPH)

-Placenta adheres to myometrium instead of separation. -Related to prior uterine surgery (similar to adhesions) -*Serious issue-obstetrical emergency* May be identified in early pregnancy via USN, plan in place at delivery helps, MRI can confirm. -An accrete may be managed without c-hyst -Percreta and increta almost *always* requires c-hyst -Future pregnancies at high risk for recurrence, rupture, hyst, and previa (placenta lying low in uterus, next to or covering cervix) *Normal Placenta* - Separated from uterine wall by fine fibrinous layer *Placenta Accreta* - Placenta attaches firmly to uterine wall lining (may be managed w/o c-hyst) *Placenta Increta* - Placenta invades at least halfway through uterine wall: have to remove uterus *Placenta Percreta* - Invasion through the uterine wall, sometimes into nearby tissues, like the bladder

Describe the clinical presentation of a patient with a breast abscess.

-Same s/s of mastitis plus localization area, same etiology/pathogen -Suspect when not improving w/in 24-48h of meds or palpable mass -Can see on USN

Given a patient in the later post-partum stage, identify concerns of post-partum depression and develop appropriate therapy. post-partum depression= PPD

-Screening of mental health should start at the first visit -Women with a hx of depression have markedly increased rates of post-partum depression/psychosis -At the first post-partum visit, we use the Edinburgh Postnatal Depression Scale (EPDS) CDC: 11-20% of women will have PPD. 600,000 annually will develop PPD. This only counts *live births* -Only 15% of these women get treated -Maternal depression in the first 18 months of life greatly affects the cognitive development of child -Ask about PPD in a positive and open manner. Offer help. *Treatment* -Patients need individualized treatment plans -Counselling should be a staple of these plans -Do not be afraid of medication -Many medications can be used in breast feeding (pump and dump, peak times) -Patients need supportive environments -Absolutely need *close follow up* *PPD Medications* -Generally paroxetine or sertraline -Citalopram also acceptable -Usually undetectable in serum of infants (try to avoid fluoxetine, long half life allows it to build up in blood of infant) -If mom is not breast feeding all meds open option (still consider adverse effects plus sleep-deprivation) -In partial responders, do add-on therapy rather than switch meds (SNRI, atypical like buproprion, or tricyclic *PPD Alternatives* In women who do not respond to psychotherapy or medications or who have severe psychosis: -Electroconvulsive Therapy: this is proven to be safe to infant, fast and effective, well-tolerated

Describe the pathophysiology of endometriosis.

-The exact cause is unknown but the leading theory is *Retrograde menstruation* due to an outflow obstruction that causes back flow through fallopian tubes followed by *Stem cell theory* -General idea is that endometrial cells get displaced into the peritoneum (ectopic location) and then they develop on Nerves, Fallopian tubes, ovaries, and pretty much anywhere they want to grow. -The biggest factor currently known for developing endometriosis is low BMI

When do you use a breast MRI?

-To determine extent of breast cancer -Not routinely performed in new cases -Best for evaluating response to treatment -Also for women with metastatic disease without origination

Summarize the normal phases of the menstrual cycle.

-Usual cycle (day of ovulation): 21 (7), 28 (14), 35 (21) -Tend to ovulate *14 days* before start of next period -Normal cycle length: 21-25, bleed for 2-5 days The following is straight from the text, I think this version is easier to understand. Pieces in parenthesis are from lecture. -Ovarian function in a normal menstrual cycle is divided into follicular phase (pre-ovulatory), ovulation, and luteal phase (postovulatory) *FOLLICULAR PHASE* 1) FIRST, Hypothalamic-pituitary-ovarian axis must be functional for normal menstrual cycle. 2) The *hypothalamus* releases pulses of GnRH into the hypothalamic-pituitary portal tract @ specific *frequencies* and amplitude. (Pulses in beg of cycle are fast and slow down after ovulation) 3) GnRH stimulates the synthesis and secretion of *LH and FSH* from anterior pituitary 4) LH and FSH enter peripheral circulation and act on the *ovary* to stimulate both follicular development and ovarian hormone production (estrogens, progesterones, androgens aka gonadal steroids) 5) Gonadal steroids are typically inhibitory at both pituitary and hypothalamus. HOWEVER, development of mature follicle creates a rapid rise in estrogen levels which acts positively to generate LH surge. THIS IS ESSENTIAL FOR OVULATION. (LH surge casuses increased production of proteolytic enzymes and protaglandins. Follicle size rapidly increases by movement of fluid into the follicular space- ovulatory follicle around 28-32mm in diameter. Via gap junctions, LH surge causes granulosa cells to induce factors to advance to oocyte from phrophase of Meiosis I to metaphase of Meiosis II.) SO, NOW following *OVULATION*, 6) LH stimulates luteinization of the granulosa and theca cells, which had surrounded the mature oocyte to form *corpus luteum*. 7) The corpus luteum continues to produce estrogen but also secretes *high levels of progesterone*. (CL expands by breakdown of the basal lamina, hypertrophy of cells, and angiogenesis to supply the precursor (LDL cholesterol) -New enzymes appear for switch back to progesterone production) 8) The thickened, *proliferative endometrial* lining produced by high circulating estrogen levels during the follicular phase is now converted to a *secretory* pattern by this luteal progesterone. -IF pregnancy occurs, the corpus luteum is "rescued" by hCG secretion from early placental trophoblasts. (Progesterone is vital to the maintenance of pregnancy (gets lining of endometrium ready so it becomes secretory -> meaning supplies/secretes nutrients)) 9) IF pregnancy does not occur, then progesterone and estrogen secretion ceases, the corpus luteum regresses, and the endrometrium sloughs = "progesterone withdrawal bleed" More on step 9 -Luteolysis- from lecture: -after 12-14 days is initiated in the absence of hCG -associated with decreasing progesterone, increased PGF-2 alpha and apoptosis of thecal-lutein cells. -CL becomes the corpus albicans with collagen deposition and fibroblast proliferation. Eventually this site undergoes remodeling. -Remember, GnRH Pulses go back up once CL goes away and negative feedback is lost.

Discuss the etiology of mastitis.

-Usually due to a Staph infection (usually meth resistant) -can be strep or e. coli -Source of organism is almost always infant's nose and throat -Develops in 1/3 of breast feeding women -Rare outside of that population

Given a patient who is unresponsive to the medical treatment for the post-partum hemorrhage, develop a surgical plan.

-Uterine Artery Ligation (works for lacerations from c/s or hysterectomy) -Uterine Compression Sutures (works for atony) -Internal Iliac artery ligation (hard to do and not very successful) -Angiographic Embolization (use if surgery is a bad plan, moderate success) -Pelvic umbrella pack (used as a last resort) -Hysterectomy

OCC

-Women who have sex with women have rates of HR HPV positivity, abnormal cervical cytology, and high-grade cervical neoplasia similar to those of heterosexual women, but undergo cervical cancer screening less often. (they should be screened the same)

*H.E.L.L.P Syndrome"* (not a specific objective)

-life-threatening pregnancy complication usually considered to be a variant or complication of pre-eclampsia HELLP stands for: *H*emolysis, *E*levated *L*iver enzymes, *L*ow *P*latelet count *if this syndrome develops= DELIVER ASAP* 3 classes: just know there are 3 classes and one is the worst. Don't memorize the class numbers (1--->3; one=omg) I: severe thrombocytopenia (PLTs < 50K) **class 1= worst** -->can cause signif. bleeding events II: moderate thrombocytopenia (50K to 100K) III: AST >40, mild thrombocytopenia (100K-150K) sequelae: 25% mortality !!! mom dies from liver rupture or CVA

Contraindications to HRT

1)pregnancy. 2)undiagnosed abnormal vaginal bleeding (might be cancer) 3)active thromboembolic disorder (DVT, Stroke, MI etc) 4) suspected or active breast or endometrial cancer. 5 )active liver disease with LFT changes 6) NO UNNOPPOSED ESTROGEN IF THEY HAVE A UTERUS

Construct a plan for prevention for each of the major risk factors for *pre-term birth*.

1. *Cerclage*- sutures in the cervix to prevent insufficiency (sewing the door closed) w/ non-absorbable sutures -can be done prophylactically or for rescue (often in wk 16 - 26) -indications: -US shows short cervix or cervical incompetence/thinning -hx of PTL/PTB or hx of cervical incompetence 2. *Progesterone* -injxns or vaginal progesterone

Parturition Phase 1

1. *First 30ish weeks* of pregnancy 2. *Uterus mainly quiet* until late stage 1 3. Begins to have some low level contractions (*Braxton Hicks toward end*) 4. *Cervix undergoes softening* a.Remains firm BUT becomes more compliant b. Due to increased vascularity and stromal (tissue) hypertrophy of cervix occurs c. Interesting: if you have collagen issues, then then you might have issues with your cervical stability ex: Ehlers-Danlos, Marfans

Parturition Phase 3

1. *LABOR* happens here 2. *Stage 1:* Contractions are sufficient enough to bring about cervical *EFFACEMENT* (streches, gets thinner) and *DILATION* (opens up) 3. *Stage 2: BEGINS when DILATION* is complete and *ENDS with DELIVERY* 4. *Stage 3:* Begins with delivery of placenta LOOK AT THIS PICTURE AND HOW FREAKING WIDE THE CERVIX GETS. OMFG.

Summarize the Phases of Parturition (Technically Parturition isn't in the objectives, but it's half the PPT and very involved with the stages of labor, which you do need to know)

1. *Phase 1: Uterine Inactivity and cervical softening* 2. *Phase 2: Preparation for Labor* 3. *Phase 3: Labor (3 stages of labor within this phase)* 4. *Phase 4: The puerperium* 4 Phases of parturition: series of physiological changes that aid in the process of labor. Actual labor occurs in the 3rd phase of parturition

Describe the use of emergency contraception.

1. *Progestin, Combination estrogen and progestin, and antiprogestin* - Mechanism of action: Inhibit or delay ovulation, prevent implantation from mucus thickening - Effective only *before* a pregnancy has been established - Must be taken within 5 days of intercourse (ideal is 3) Side effects • Nausea (less with progestin only) • Irregular bleeding • Breast tenderness • Abdominal pain • Headache • Fatigue Regimen - *progestin only* •levonorgestrel* - (Plan B and no Rx needed) single/two dose protocol (12 hours apart) - *Combination estrogen and progestin* •estradoil and levonorgestrel in 2 doses 12 hours apart (use antiemetic too) - *antiprogestin* • progesterone antagonist, requires Rx, Ulipristal (RU486, not in US, in high doses this is the early pregnancy termination drug, very expensive, very few providers have permission to prescribe) *Progestin only should be 1st line as it is more effective than estrogen-progestin combination* *Non Hormonal Option:* *Copper IUD* - Use up to 5 days after unprotected intercourse - *failure rate of 1% (the best rate)* - Advantage in that can then be kept in and then used for long-term contraception This old photo always makes me smile. BACK YOU STORK DEVIL. BACK.

Parturition Phase 2

1. During last 6-8 weeks of pregnancy 2. If this phase occurs earlier or later, it can cause preterm or late labor 3. Changes in myometrium stimulate more painful contractions (uterine irritability), top half becomes firm, lower half becomes soft a. This happens because of contraction associated proteins b. Fetal head also able to descend through pelvic inlet: dropping, aka LIGHTENING (2-4wks before labor in nulliparas) After dropping SOB and heartburn may improve, but pressure on bladder is worse. Stomach looks different 4. Cervix goes from softening to ripening (more aggressive hormones)

Given a clinical scenario, select the appropriate therapeutic option for a patient experiencing a first trimester abortion.

1. Expectant management of spontaneous incomplete abortion has failure rates as high as 50 percent. 2. Medical therapy with prostaglandin E1 (Misoprostol) has varying failure rates of 5 to 40 percent. 3. Curettage usually results in a quick resolution that is 95-100% successful. It is invasive and not necessary for all women.

Gravid/Para Examples (G_P____) 1. Never had a child, never been pregnant 2. Pregnant once, had an abortion at 11 weeks, no children 3. Pregnant now for the first time 4. Sandra delivered a baby boy at 38 weeks gestation, twin girls at 36 weeks, and had a miscarriage at 11 weeks. 5. Ms. Mahler had three infants delivered full term, one infant delivered preterm, one abortion and four currently living children. 6. Ms Navi is a pregnant client who states "I had a son born at 38 weeks, a daughter born at 30 weeks who died from cancer at age 10, and I lost a baby at about 8 weeks"

1. G0P000 2. G1P0010 3. G1P0000 4. G3P1113 5. G5P3114 6. G4P1111

Parturition Phase 4

1. Occurs in post partum period 2. Myometrium stays contracted (compressing vessels) to prevent hemorrhage 3. Uterus involutes and cervix repairs itself 4. Lactation begins (Milk begins to "let-down" into mammary glands 5. Ovulation returns (depends on lactogenosis) 6. Stage 4 can take 4-6 weeks

Normal menstrual cycle

28-35 days Last up to 5-7 days

Summarize the types of first trimester abortion (OVERVIEW).

3 Broad Categories of Abortion: 1. Spontaneous abortion—this category includes *threatened, inevitable, incomplete, complete, and missed abortion*. - Septic abortion is used to further classify any of these that are complicated by *infection* due to products of conception remaining. 2. Recurrent abortion—identifies women with repetitive spontaneous abortions so an underlying factor(s) can be treated 3. Induced abortion—surgical or medical termination of a live fetus that has not reached viability. *Bleeding is by far the most predictive risk factor for pregnancy loss* *Think about RhoGAM for all of these!* 1/3 of women will have a 1st trimester spontaneous abortion -usually no symptoms, something causes fetal death and you have spontaneous delivery Most common cause of 1st trimester spontaneous abortion is *Karyotype abnormality* Most likely cause in 2nd trimester is *anatomical abnormality *

Labor Stage 1

4-6hrs *CONTRACTIONS* (a) *PAINFUL* contractions at *REGULAR* intervals that *CAUSE EFFACEMENT and DILATION. * (b) Labor hormone is *oxytocin*. It is what they use to induce labor if needed. (c) Contractions can begin forceful and suddenly or for others is begun by release of mucus plug (new least fav word) from the cervical canal (d) Contractions slowly *shorten from q 10 minutes at beginnings of Stage 1 labor to as little as q 1 minute in the 2nd stage*. and increase with severity. (e) *Constant contractions are BAD* for the baby (blood flow compromise) (f) Upper and lower uterine segments even more divided. Can even feel *firmer top and softer bottom of uterus.* Important, because if it was all firm muscle contracting, baby would go nowhere. (g) Uterus makes a tube that will help expel fetus, lower soft sections have less tone *EFFACEMENT* (see photo, happens faster second time around) (i) Effacement occurs when the cervical length shortens from *2 cm thick to thin edges* 1. This is discussed in percentages 2. *Goal is 100%* *DILATION* (ii) Dilation occurs during contractions when the LUS (loweruterine segment) and cervix pull apart 1. This is discussed in cm, pointer finger= ~1cm wide. Make your widest peace sign = ~10cm 2. *Goal is 10* 3. Fetus will typically descend into place as you dilate. Slow if 1st time, quicker if not 1st time. 4. *Fully dilated means end of Labor, Stage 1* 5. No proof in massage of mucous plug (idk what this means, it was in the PPT)

Describe a functional ovarian cyst in terms of pathophysiology.

A functional cysts is a corpus luteum or follicle that has enlarged and can possibly rupture. -If the sac closes up after release of egg it is called a luteal cyst -If the egg does not release then they can enlarge and rupture (follicular cyst) They are termed Functional because they form during or after ovulation (they are common and only become a problem if they enlarge or rupture) Usually resolve on their own and are asympomatic You will see free fluid on US As long as cyst is under 5 cm it is not a problem 5-10 cm you increase risk of torsion but just observe. If they are symptomatic then OCP may help speed the resolution Because they are sensitive to hormones.

Hallmark of endometriosis

A hallmark of endometriosis is sudden and lasting pain with intercourse after years of pain free intercourse

Summarize the PALM-COEIN system for the causes of AUB.

A pneumonic for remembering the causes of AUB other than pregnancy! *P*olyp *A*denomyosis Sx Tx (worse form of endometriosis where it consumes myometrium) (if you have pain Sx like endometriosis + HMB then be suspicious of this) *L*eiomyoma (fibroid) *M*alignancy *C*oagulopathy (bleeding disorder) *O*vulatory Disorders *E*ndometrial dysfunction (PCOS) *I*atrogenic *N*ot yet classified (we don't know)

Summarize the ASCCP recommendations for abnormal PAP smear results: AGUS

AGUS (atypical glandular cells of undetermined significance) *HPV testing not needed for this group* - Requires evaluation of the endometrium - Usually a *squamous neoplasia* - Elevated risk of breast and colon cancer *Colposcopy* to look for other GYN cancers

Review uterine leiomyomatas and differentiate these from uterine cancer.

AKA myomas or fibroids They are very common round rubbery benign tumors on or in uterus *These can be easily removed from the uterus, They unlike uterine cancer do not invade deep uterine tissue* -Sensitive to hormones and shrink during menopause -If they necrose and die they become painful *Sx:* -AUB -dysmenorrhea -Dyspareunia -Constipation *Imaging* -CT *Tx:* -May pass on its own. -for pain use NSAIDs and COC *Surgical options (if needed)* -*Uterine artery embolization* (cut off blood supply) after this you MUST have patient on BC because they can no longer support pregnancy -Myomectomy (remove fibroid in small procedure, uterine sparing) -Hysterectomy

Summarize the ASCCP recommendations for abnormal PAP smear results: ASCUS

ASCUS - Extremely common If HPV is present & low risk --> *cotest in 3 years If HPV is present and high risk --> colposcopy (Risk- Cervical Ca risk increases with older age, obeisity, smoking, *Multipara, Immunosuppressed*)

Compare and contrast acute and chronic PID.

Acute: Majority of cases -Rapid onset -Vague lower abdomen symptoms -Exam shows lower genital tract inflammation Chronic: -Caused by chronic inflammation (might not be any infection any more) -Rare -low grade fever -Weight loss -Often associated with Actinomycosis (rare bacteria) and TB

Colposcopy Basics

Allows for further visualization and diagnosis of the vagina and cervix 3 major components: 1. Cervical visualization 2. Squamo-columnar junction visualization 3. Transformation Zone Classification *Reid Colposcopic Index* - grades lesions on these factors 1. Margin 2. Color 3. Vascular Pattern 4. Lugol Solution Staining Scored from 0-2 *A higher score is a bigger, badder lesion*

Risk of consumptive Coagulopathy*

Also called Disseminated Intravascular Coagulation (DIC) -Perinatal mortality is high -Pregnancy alone increases the clotting rate, but at the same time decreases fibrinolysis -In certain events, thromboplastins are released and coag factors/platelets are depleted enough to cause DIC (like in placental abruption or prolonged hemorrhage)

Develop an initial approach to the workup of PID including laboratory testing and imaging.

Always check if pregnant first! -possibly be an ectopic -do pelvic exam -CMT --> culture for GC/CT -Adnexal tenderness --> Admit/US -ESR/ CRP often elevated -Screen for other STDs -Wet prep will show leukocytes, possibly show BV signs (clue cells, smell) -If US does not show anything then CT can be ordered

Background info: Basic anatomy of the Amniotic Sac and placenta

Amniotic sac has two membranes: - Chorion = outer membrane against the uterine wall. - Amnion - Inner membrane by the baby

Attenuation of perineal body Bulging rectocele Isolated Rectocele Sliding enterocele (rare)

Attenuation of perineal body -top left (ouch) Bulging rectocele- Top right Isolated Rectocele- lower left Sliding enterocele (rare)- lower right (can interfere with BM)

Determine how to manage cervical intraepithelial neoplasia (CIN).

BASICS OF CIN: (cervical intraepithelial neoplasia) - Associated with persistent HPV infection - Cervical cancer usually follows CIN Risk factors for CIN include: - Multiple sexual partners - Early onset sexual activity - Tobacco abuse - HPV infection CIN 1: High rate of spontaneous regression NOT treated aggressively Can be observed indefinitely *Treat ONLY if persists more than 2 years* *DO NOT TREAT in a woman under 25 (pap and colposcopy every 6 months)* CIN 2 & 3: (30% of CIN 3 will progress to CA) *Ablation or excision* - Ablate if: found on colposcopic sampling in small are - Excise if: found in endocervical sampling, colpo is inadequate, or if it is recurrent - If under 25: observe closely (cytology and colposcopy every 6 months) - If pregnant: delay until after childbirth unless cancer present. Ok to do colposcopy *DO NOT BIOPSY ENDOCERVIX IF PREGNANT*

Treatment of ectopic pregnancy repeat, but straight from powerpoint

Based on clinical presentation and diagnostic findings -If ruptured, surgery is only option -Remove ectopic v remove tube? -If large, surgery is strongly recommended -Small (less than 3.5 cm) and stable ectopics can receive methotrexate injection -Patients MUST be COMPLIANT and motivated to have f/u testing -Need to have HCG of less than 5000 for best success -Repeat levels at 0, 4, 7 days and need 15% decline every time

Discuss pathophysiology of adnexal torsion.

Basically the fallopian tube +/- the ovary gets twisted -most commonly its both -Usually associated with a cyst or mass > 5-6cm -can be ovary and fallopian tube together, ovary turns by itself, fallopian tube turns by itself ( listed in order high to low chance) Benign masses are most likely to turn. -Almost ALWAYS R side When either the ovarian or uterine vessels becomes occluded it -blood flow in (high pressure- arteries) continues -blood flow out (low pressure- veins) does not -leads to edema then infarct... Can be salvaged if you operate quickly (time equals ovary)

Given a clinical scenario, develop a treatment plan for a patient with amenorrhea.

Basically this all depends on what is going on. If they have eating disorder solve that. If they have tumor solve that. If they have PCOS treat that. If they don't want to get pregnant who cares! All hypogonadic patients can receive estrogen replacement (give a ethinylestradiol/ progesterone) (assuming no contraindications) NEVER give estrogen by itself (endometrial cancer)

OCC

Below from study guide *Primary PPH* (w/i 24h of delivery): Retained placenta, uterine atony, cervical/vaginal lacerations, coagulation defects, uterine rupture, uterine inversion The "4 Ts" - *T*one, *T*issue, *T*rauma, Clo*t*ting *Secondary PPH* (>24h - 12 weeks postpartum) -Subinvolution of placental site, retained products of conception, infection, coagulation defects -Post-partum psychosis -Chorioamnionitis (a risk factor of post-op c/s infection) is a complication of pregnancy caused by bacterial infection of the fetal amnion and chorion membranes. -Postpartum uterine infection or puerperal sepsis has been called variously endometritis, endomyometritis, and endoparametritis. Because infection involves not only the decidua but also the myometrium and parametrial tissues, we prefer the inclusive term metritis with pelvic cellulitis

Summarize the ASCCP recommendations for abnormal PAP smear results: BETHESDA SYSTEM NOMENCLATURE

Bethesda System Nomenclature - Based on adequacy and epithelial cell abnormalities (presence of transformation zone and adequate sample) Abnormal Findings: Atypical Squamous Cells of Undetermined Significance (ASCUS) Low Grade Squamous Intraepithelial Lesion (LSIL) High Grade Squamous Intraepithelial Lesion (HSIL) Atypical Glandular Cells of Undetermined Significance (AGUS) Carcinoma Infections

If a patient pregnant patient presents with bleeding, what do you get?

Blood type!

Where do breast cancers like to metastasize?

Bone, lung, liver, brain

Café au lait spot.

Café au lait spots, macules, are flat, pigmented birthmarks. more than 1 is concerning

Compose a diagnostic work up for a patient suspected of having uterine cancer.

Can try an EMB- but its not that sensitive (25%) CA-125 elevated level can clue you in *CT*: Sarcoma! see multiple layers MRI: may help distinguish between a benign entity and a scary one, but won't tell you which one. *PET* used for staging once diagnosis is made

Summarize the physical symptomatology that accompanies the physiologic changes listed in #2 above. (perimenopause, menopause, and post-menopause.) *Vasomotor Sx* (AKA hot flashes/night sweats)

Casue: *Peripheral vasodilatation*, but no one knows exactly why, thought to be due to disfunction of the *thermoregulatory nucleus of the hypothalamus* due to *estrogen withdrawal or rapid fluctuation* *Norepinephrine* (lowers thermoregulatory effects) and *Serotonin*(mediates heat loss) Estradiol lowers alpha adrenergic receptors and increases norepi and serotonin, this change in Norepi and serotonin change the body's temperature set point and all of these subtle changes trigger hot flashes. Sx: Skin temp rises 10-15 degrees C. Sweating begins on upper body and spreads Increase in both awake and sleep SBP and HR. May have palpitations, anxiety, irritability, and panic. Lasts 1-5 minutes But may take up to 30 minutes to return to normal skin temp. Onset: 2 years before FMP Most women will have these severe sx for up to 5 years after FMP 1/3 of symptomatic women will have moderate to severe symptoms for at least 10 years Sx assoc with: low exercise, smoking, weight, depression, lower socioeconomic status

Discuss the causes of post-partum hemorrhage (PPH).

Cause is almost always found and treatable. Causes include: 1. *Uterine atony* -> no contraction of uterine corpus, which normally contracts promptly after delivery of placenta and constricts spiral arteries/stops bleeding. -Most frequent cause 2.*Bleeding from placental implantation site* (Placental adherence syndromes: Accreta, Percreta, Increta) See next card. 3. Genital tract trauma 4. Combination of above 5. *Infection*: also called endoparametritis, endomyometrisis -Delivery route is biggest risk factor, highest is in *C-Section*... unless there's prolonged rupture of membranes, prolonged labor, multiple cervical exams -Untreated antepartum infections (GC/CT) can also be a cause, but it's usually *multi-pathogenic* -Post partum fever is a *big warning sign* -Treat with *antibiotics*, do not be hesitant, don't wait on cultures *How do we determine source?* -Careful physical exam -Is the uterus boggy? Soft? (Atony= boggy and soft) -Do you have clots during massage? (Atony) -Do you see obvious laceration? (Genital tract trauma) -If not, consider uterine laceration (In some cases, blood accumulates in intraperitoneal space and hemorrhage will not be identified in timely fashion.)

Describe the screening for cervical pathology: PAP SMEAR

Cells scraped off cervix then examined Pap smear is very specific, but low sensitivity Pap test is not the best at detecting adenocarcinoma, which is increasing in prevalence Pap smear guidelines: *Start at age 21* Screen every 3 years until age 30. At age 30-65 can either do: Cytology alone every 3 years OR Cytology plus HPV testing (co-test) every 5 years Can stop at age 65 if: *3 consecutive neg paps alone or 2 negative co-tests* Any woman with prior treatment for a lesion should continue testing for *20 years past lesion* *HIV positive women continue annually* *Cytology plus HPV testing is the PREFERRED strategy from all recommendation groups* Times to avoid pap smear: - Active menses - Within 24 hours of douching, sex, or intravaginal meds *Do not:* - Wait until all unexplained discharge or abnormal uterine bleeding (AUB) is gone (signs of cancer!) - Touch the cervix before doing the pap (may remove epithelial cells you need) *You can remove discharge with a large q-tip (allow to absorb from the corner)

Summarize the guidelines for use of the HPV vaccines.

Cervarix: bivalent (HPV 16 and 18) Gardasil: quadravalent (HPV 16,18, 6, 11) Gardasil 9: same as above plus 31,33,45,52,58 Belief that percentage protected against will increase from 65% to 80% *All are 3 shot series given in 6 months* Guidelines: (according to ACOG; not in PowerPoint) *Target age is 11-12 years old* for males & females Recommended up to age 26 if not received earlier

Discuss the etiology of chronic pelvic pain.

Chronic pelvic pain is: -non cyclic pain for 6 months -Pain that localizes to pelvis, abdomen below T10, LS or buttock region -Bad enough to cause functional disability or need medical Tx The most common causes: 1. Endometriosis 2. Symptomatic fibroids 3. IBS Important to get a Detailed history. Surgery causing adhesions and sexual abuse are important don't misses in a history.

Summarize the clinical presentation of a patient with uterine cancer.

Clinical presentation: -*Abnormal vaginal Bleeding* -Pelvic pain -Abnormal necrotic vaginal discharge with foul smell -Enlarged uterus (usually RAPIDLY enlarging) -Prolapse of polyps -May have GI/GU complaints -pt doesn't have to have all of these. Visceral deep ache, gained weight centrally (feels pregnant), constipation, frequent urination,

Summarize the types of first trimester abortion (Complete)

Complete Abortion - pain, cramps, & bleeding subside - pre-pregnancy uterus size - *open cervical os* (if the abortion was days ago then the os will be closed) - all POC are expelled - *no FHT* - pregnancy not salvageable -verify with HCG levels -US to make sure all POC are gone, Give Rhogam if Rh- mom

Identify a cystocele, rectocele, and uterine prolapse

Cystocele- Bladder prolapsed into vaginal vault Rectocele- Front wall of the rectum bulges into the vaginal vault Uterine prolapse- uterus slides downward into vagina, 3rd MC indication for hysterectomy

Given interpretation of a bone mineral density test, compose a treatment plan for an osteopenic patient. Also Construct a lifestyle/modification plan for patients with osteoporosis/osteopenia.

DEXA= *-1.0 to -2.5* is osteopenia Part of treatment is lifestyle modification in the form of *Prevention*: Weight bearing activity daily 1200 mg calcium and vitamin D 800-1000 IU daily No smoking, limit EtOH Assess fall risks Measure height annually (shrinking= fractures in spinal cord) DEXA at 65 Add *medications* for osteopenia: HRT Can reduce fractures by 50% when started *at or soon after menopause* Continued *long term* disease rates decrease 33% fracture reduction rate at 5.6 years This benefit is lost soon after HRT discontinuation SERM (technichally an HRT) *Raloxifene* (can cause clotting) Activates Estrogen receptors *in the bone* but *not in breast or uterus* Very protective against *vertebral fractures* CONTRAINDICATED in patients with h/o clotting

Normal 28 day cycle broken down

Day 28-Day 7 (you bleed) Withdraw of progesterone makes endometrium bleed. (period) When estradiol/ progesterone low tells GnRH to tell Ant Pituitary to start pulsing LH/FSH (mainly LH during the follicular phase) Theca cells make Testosterone and small amount of progesterone in response to LH FSH converts via aromatase into estradiol Estradiol thickens endometrium in preparation to receive implantation. Day 7-14 Goal is maturation on dominant follicle Estradiol (E2) is elevating and causing GnRH to drop off Ant. Pituitary no longer releases FSH Ant. Pituitary releases LH surge in response to Estradiol Huge Estradiol increase Follicle continues to release progesterone (only release moderate amount though) End of (proliferative/follicular phase) Day 14-21 (Start of luteal phase/ secretory phase) Release of dominant follicle ooctye (egg) is dropped into fallopian tube The Corpus Luteum continues to mature and release progesterone Day21-28 Progesterone level very high (produced by CL) Controls thickness and improves vasculature in endometrium If no hCG then progesterone level drops Estradiol level drops

Given a patient with PCOS, compose an initial therapeutic plan.

Depends on their goal: *Conservative management:* -weight loss (10% helps huge!) -Low carb diet (for the betus) -supportive control (acne meds, razors, etc) *Don't want to get pregnant:* -Oral contraceptive (yas or something with a progesterone that reduces androgenic side effects) -+/- Metformin for the Betus -+/- Spironolactone *Don't want to get pregnant or take birth control pill:* -Put them into withdraw with Provera shot every 2 months *If they want to get pregnant:* -Lose weight (very effective) -Clomiphene citrate (stimulates FSH) -Metformin (for the Betus) -Bromocriptine (would treat an elevated PRL) - IUD is a good option! Thins cervical lining and chops off the heads of sperms (lecturer did say this! )

Summarize the 4 stages of prolapse identified during the physical exam.

Depends on where the Apex (cervix) is at Baden-walker

Given interpretation of a bone mineral density test, compose a treatment plan for a patient with osteoporosis.

Dexa= * at or below -2.5* or at risk pts who have *Any hip or vertebral fracture* *Treat All women with a score -1.0 to -2.5 who have a risk factor for fracture due to:* Hx of Falls Balance problems Visual impairment On meds that limit/impair vision With arthritis Alcoholics Tx: Same as osteopenia, but you add *Bisphosphonates* (Fosamax, Binostro, Actonel, Atelvia, Boniva, and Reclast) and SERMS are only used for mild to moderate osteoperosis UNCOMMON Risk factors of Bisphosphonates: *osteonecrosis of the jaw (ONJ)* (worse around dental procedures) *Atypical femur fracture* (after 5 years of treatment) To reduce risk factors consider drug holidays for long term pts or before a dental procedure Once you add bisphosphonates, make sure to *recheck DEXA every two years*. Oral drugs have poor bioavailability and can cause GI issues, but give the bisphosphonates orally if tolerated then IM or IV. Then if they can't take bisphophonates or they are not working, move up to *teriparatide*, (low dose PTH that is the *only drug that builds bone up*, others just prevent reabsorbtion)

Given a clinical scenario discuss the workup for a patient with a breast cyst.

Diagnosed on ultrasound 40-50 years old=MC 3 types Simple cyst ------------------ -Sonolucent, smooth margins -treat only if painful (treat by aspiration) -If symptomatic and recurrent, excise for best treatment -Otherwise can reimage or continue to repeat aspirations Complex cyst -------------------- -Have internal echoes on ultrasound, which is debris (looks like honeycomb) -May look like a solid mass with fluid on top of it -Treatment is usually aspiration -Culture any purulent fluid -If this does not resolve, then core needle biopsy is performed Complicated cysts ----------------- -Will also have internal cystic masses, not much fluid -These can actually be a carcinoma -Can aspirate via ultrasound -This can make surgical localization difficult -Excision is generally recommended

Compose a treatment plan for a patient with mastitis.

Dicloxacillin (can start empirically), erythromycin (for penicillin sensitive pt), or vancomycin (suspect resistant organisms) Most recommend that milk be expressed from the affected breast onto a swab and cultured before therapy is begun. Pt. can keep breastfeeding- stopping leads to engorgement, increases abscess formation. Start feeding on unaffected side. May need to pump.

Construct an initial work up plan for a patient with a disorder of development. Asked Dr. G about this on Piazza. -Not covered in lecture, chapter 16

Disorders of development: puberty either too early or late. *Early* Work-up is dependent on which symptoms are seen. -In both (central/peripheral) we get: *bone age/FSH, LH, estradiol* -In central we get: TSH, pelvic sonogram (through abdomen NOT TVUS), and a brain MRI -In peripheral we get: androgen testing to include DHEAS, testosterone commonly, and maybe other random ones -If you still can't tell: hormone test with Lupron Peripheral: Testing of girls with peripheral precocious puberty finds estrogen levels that are characteristically elevated, whereas serum levels of LH and FSH are low. Bone age determination shows advanced aging, and GnRH stimulation shows no elevation in serum LH levels. Central: In affected girls, advanced skeletal maturation is seen. In addition, serum FSH, LH, and estradiol levels are elevated for chronologic age and typically lie in the pubertal range. In either *peripheral precocious* or *central precocious* tumors that secrete exogenous hormones can be the cause. These can be found in brain or ovaries (far more common in central). *Delayed Puberty* constitutional, anatomic Ex: Patient is at or around 13 yo and has no secondary sexual characteristics or no menses by age 16. This was the commonly called "late bloomer." A *constitutional delay* (physiologic) patient is the *exception* to a workup being needed. You can do a trial of very very low dose estrogen only on these folks until puberty progresses (note: not 3-5 years or more), usually kicks in a year or so, remember we talked about it taking up to 2 years to become regular in early menarche. In that time period, people have unopposed estrogen buliding up and it is not enough to do any damage. We have *anatomic defects* as a separate category which can be congenital or acquired. These patients will not have a delayed puberty in that they develop secondary sexual characteristics but they will meet the second criteria for delay-no menses by age 16 because they menses cannot leave the body in the case of the congenital defect. All of the other disorders from here on out are the hypergonadotropic hypogonadism/hypogonadotropic hypogonadism/eugonadotropic eugonadism spectrum. -Investigate whether pubertal tanner stages have been reached and whether menses have begun (cycle, duration, amt. of flow, changes), surgical history, family history, social history -ROS (sx relating to thyroid dz, pituitary or CNS tumors, Hirsutism, pelvic pain...) -Physical Exam: general appearance/BMI, signs of turner syndrome, midline facial defects (ant pit gland defect), pubic hair pattern, rectal and digital vaginal exam (help identify obstruction) -Testing: Always get pregnancy test. -Could give exogenous progesterone and monitor for progesterone withdrawal bleed, -Serum hormone levels (LH and FSH can help confirm hypogonadotropic hypogonadism) -Prolactin and thyroid-stimulating hormone levels are tested in most patients with amenorrhea as prolactin-secreting adenomas and thyroid disease are relatively common and require specific treatment. -Serum testosterone levels are measured in women with suspected PCOS or with clinical signs of androgen excess. -Serum dehydroepiandrosterone sulfate (DHEAS) production is essentially limited to the adrenal gland. -Chromosomal analysis, radiologic analysis Basically, (figure 16-7) If they have a uterus and are not pregnant, see if TSH (thyroid problem), prolactin, or FSH is elevated. If FSH is elevated ->Gonadal failure If FSH is decreased-> eating disorder? No, get MRI to check for tumor. If FSH is normal -> -Check testosterone -> If inc get u/s -DHEAS-> if inc get adrenal MRI for tumor -17-OHP-> inc = Congenital adrenal hyperplasia

Describe the mechanism for dizygotic twin pregnancy formation.

Dizygotic = Two separate eggs fertilized by two different sperm 2 options for placenta and the membranes (chorion and amnion): - Two placentas, Dichorionic, Diamniotic (Separate rooms & beds)(25 - 35%) - Fused placenta, Dichorionic, Diamniotic (Separate rooms & beds)

Questions to ask patient with endometriosis?

Do they want kids? do they want to end pain or end fertility?

Compose a differential diagnosis list for a patient presenting with bleeding in the first trimester.

Ectopic Pregnancy Early pregnancy complications (abortion) Placental polyp Hemorrhagic corpus Luteal cyst Idiopathic bleeding in a viable pregnancy Cervical abnormalities Infection of the vagina or cervix Subchorionic hemorrhage Vaginal trauma Molar pregnancy "never normal, but not uncommon, can be idiopathic"

When do you do Egg quality testing?

Egg quality testing -over 35 -1st degree relative with premature menopause -previous ovarian insult -smoking -Planned IVF -Oncofertility

Given a clinical scenario, compose a treatment plan for a patient with chronic pelvic pain.

Endometriosis: -High dose NSAIDS -Combined OCP -GnRH Agonist (Lupron) -Opioids -Nerve destruction -TAH/TVH BSO (remove it all) Symptomatic fibroids: -OCP/ NSAIDs -Uterine Artery embolization -Myomectomy -Hysterectomy IBS- FODMAP free diet

What can provoke a blood clot ?

Estrogen : HRT This is why it's important to talk about lifestyle changes

Construct an initial workup plan for a patient with complaints of amenorrhea.

First check hCG! Then determine if its *Primary* -No menses by 13 + no signs of pubertal development or -No menses by 15 but with signs of pubertal development or thelarche (breasts) in the past 3 years *Secondary* -No menses for 3 consecutive months or -Fewer than 9 cycles per year

Describe epidemiology of twin gestations.

Frequency of monozygotic twins is relatively constant across all age and ethnic groups Increased frequency of dizygotic twins in: - African-Americans - Older women - Positive family history - Use of ovulation induction Between 1980 and 2009, the twin rate increased 76% (from 18.9 to 33.3 per 1000 births) EXPLANATION - Advanced age of mother (4.4 x) - Increased use of ART (Assisted Reproductive Technology)

Summarize hormone replacement therapy

HRT is similar to birth control, but birth control is either *Progesterone alone or combined* and HRT is either *Estrogen alone or combined*. There is a tiny bit of actual chemical difference and dosage difference , so combined forms of HRT and the combined forms of birth control *will not work* as replacements for each other HRT *treats vasomotor Sx* (hotflashes) that are the result (in part) of decreased estrogen. Also it can help *prevent bone loss* from progressing to but for safest cardiovascular results start early (around MT), and stay on it* Tell the pt that they should not be surprised if the have menstruation like bleeding when they have been given HRT Benefits: 1)Increase in BMD 2) Decrease in fracture rates 3) Decrease rates of colorectal cancer Risks: 1)Increase in clots 2)Increase in rates Estrogen receptor positive breast cancer 3) Increase in rates endometrial cancer if UTERUS intact and given unopposed estrogen (so don't ever do that)

Summarize the ASCCP recommendations for abnormal PAP smear results: HSIL

HSIL (High grade Squamous intraepithelial lesion) - Presence of HPV and CIN 2 or 3 *All of these patients go directly to colposcopy* Acceptable to do a LEEP with it (loop electrosurgical excisional procedure) Almost all HSIL lesions require treatment

Describe the workup for a patient with habitual abortions.

Habitual/Recurrent abortion - *3 or more consecutive pregnancy losses at ≤ 20 weeks* Causes of recurrent abortion: Genetic/Immunologic Causes - parental chromosomal abnormalities - antiphospholipid antibody syndrome - uterine abnormalities (Asherman) - genetic disorders of blood coagulation - thrombophilias caused by mutations of factor V Leiden, prothrombin G20210A mutation, antithrombin III, proteins C and S, and methylene tetrahydrofolate reductase Endocrine Causes - Hypothyroidism Infectious Causes - Chlamydia trachomatis and Listeria monocytogenes are associated w/ spontaneous abortion Environmental Causes - Cigarettes - Alcohol - Radiation therapy * Asherman syndrome - uterine synechiae* (uterine scarring after D&C, may interfere with next pregnancy) - usually results from destruction of large areas of endometrium. - can follow uterine curettage or ablative procedures - treatment is done using *directed hysteroscopic lysis of adhesions*

8. Discuss the diagnostic method for confirmation of endometriosis.

History will clue you in *Physical exam:* -Fixed retroflex uterus -Blue or red powder burns on cervix -Uterosacral ligament tenderness/nodules *Labs:* -Only done to rule out other issues (pregnancy, CBC pre-op, CA-125 for ovarian Cx) *Imaging:* -transvaginal US is often done first. It won't diagnose endometriosis but it may find endometriomas (chocolate cysts) (pictured) -*Laparoscopy* is the gold standard to diagnose endometriosis. It is also used to stage and remove endometrioma's and the Endometrial tissue implants.

Summarize the stages of reproductive aging according to the *STRAW* (*Stages of reproductive aging workshop*) method

How long you stay in each section, and age of onset varies from woman to woman. *Perimenopause/Climacteric*: (AKA menopausal transition) straw stages -2 to -1. These are where the big changes occur. It's usually between 42-58 yoa, and can last more than 7 years. *FMP* is the final menstrual period. After this and amenorrhea for *at least 12 months* you begin menopause Post menopause is anything after menopause begins You also don't need any labwork to determine any stage, *it is entirely symptom based* and those Sx vary from woman to woman. (also MT stands for menopause transition)

OCC Spermicide

I thought it was always liquid before this chapter... Fold in half over finger, and push it as deep as possible, you want it to dissolve close to the cervix. Max effective = 1 hour.

Describe the screening for cervical pathology: SCREENING AFTER HYSTERECTOMY

If hysterectomy was due to high grade neoplasia: *3 pap tests in first 2 years* *pap smear of cuff (top of vagina after hysterectomy) every 3 years for 20 years* NO CO-TESTING IS NECESSARY FOR THESE WOMEN (they don't have a cervix at risk for HPV) If hysterectomy done for benign reasons: *NO MORE PAPS!* If the patient still has a cervix: Proceed with regular age-based guidelines

Given a clinical scenario, correctly choose a contraceptive method from: *hormonal*, barrier, intrauterine, post-coital, or sterilization.

If it stays in the body long term, it'll be progestin only.* *Nexplanon Implant -Progestin* • Uses Etonogestrol • *Subdermal implant* • Suppresses ovulation, increases cervical viscosity, induces endometrial atrophy • Works for *3 years* • Requires training from manufacturer • Same contraindications as any other progesterone method • *Most effective method of reversible contraception* Side Effects • *Irregular bleeding- #1 reason* • Acne • Weight gain • Headache • Breast pain *Injectables (Depo) - Progestin* • Available *IM or SC* given within 5 days after menses starts -*Slower return of fertility* aafter discontinuation • By 18 months 90% of users ovulate -Concern regarding *bone loss* -Weight gain, up to 5 lbs per year -Irregular bleeding (why 1/4 will stop) *Nuva Ring - Combo progestin/estrogen* • Manually insert into vagina a small bendable bracelet-like ring • Leave in place for 3 weeks at a time, and take out for the 4th week. • Prevents ovaries from releasing eggs and thicken mucus. *Transdermal Patch - Combo Progestin/Estrogen* Ortho Evra - Inner adhesive layer, outer water resistant layer - apply to the buttocks, upper outer arm, lower abdomen, or upper torso but avoids the breasts. - vascular risk concerns *Oral Contraceptive Pills - OCPs - Progestin only or combo P&E* -See below steroid hormone sections.*

Given a clinical scenario, compose a management plan for a patient *without contraindications* to hormone therapy.

If the pt is having vasomotor Sx and *Still has a uterus* then you *never give Estrogen without progesterone* You will give them endometrial cancer. Instead do *one* of these things: 1. Combination HRT 2. Estrogen x 25 days, add progesterone x 10 days. No drugs x 5 days. 3. Estrogen continuously with a progestin for the first 10 days of the month. If they *have had a hysterectomy* then you have more HRT options that *only contain estrogen*Always start at a low dose and reasses. This is not applicable if they had endometriosis, as estrogen alone will make it flare up, so *combo HRT for women with endometriosis*, EVEN IF THEY HAVE NO UTERUS. This is an exception. You can also combine with Tx from the non HRT slides

Summarize the physical symptomatology that accompanies the physiologic changes listed in #2 above. (perimenopause, menopause, and post-menopause.) *Weight gain*

In MT, metabolism slows Fat distribution goes to the abdomen and viscera Increased insulin resistance Risk of Diabetes Risk of Heart Disease Can be negated with: Lifestyle modifications HRT helps slightly

(OJC) Why FSH and estrogen Rises after STRAW -3

In the late 40s and in early MT , FSH levels rise and cause higher ovarian follicular response, which raises overall estrogen levels. This rise in FSH is due to the fact that at this age your follicles now kind of suck at releasing inhibin, and lower secretion of inhibin means higher FSH levels.

When do you want to get a prolactin?

In the morning! especially in patients with amenorrhea

Summarize the pathophysiology of PCOS.

Inappropriate GnRH release causing a shift to predominantly LH being released (Not sure why) -LH makes Testosterone (which is why they have male pattern hair) -Because they have so much LH in their blood, they rarely have an LH surge -No LH surge Means follicle does not release an egg, and instead forms into a "cyst" -No Follicle release means endometrium builds up, and irregular periods.. no ovulation . PCOS patients have too much testosterone (hyperandrogenism) They also have increased risk for: -Metabolic syndrome (often over weight) -T2DM -Cardiovascular disease -Endometrial carcinoma (they build up a thick endometrium and don't shed it regularly)

Summarize the types of first trimester abortion (Incomplete)

Incomplete Abortion - heavy bleeding - severe cramping - *open cervical os* - some POC(products of conception) are expelled, some are still in cervix - *no FHT; fetal heart tones* - pregnancy not salvageable same management as above ~16-23 weeks -Remove any remaining POC Late abortion increases risk to mom due to uterine perforation of fetal bones

Summarize the types of first trimester abortion (Inevitable)

Inevitable Abortion - gross rupture of the membranes - *open cervical os* - almost always followed by uterine contractions or infection - usually presents with bleeding, cramping, or fever - pregnancy not salvageable - *+/- FHT* -3 options: can observe, give misoprostol, or perform curettage -Counsel on what is happening and give Rhogam to Rh- moms

Discuss the pathophysiology of pelvic inflammatory disease (PID).

Infection of Upper Genitals ( infection has spread past the cervix, into the uterus and ovarian tubes, and has a streaight shot to the rest of the peritonium) -75% caused by GC/CT -But can be caused by others (often anaerobes) -66% of pts also have BV (smelly) Symptoms: -Pelvic pain -Fever -Purulent Vaginal discharge -N/V -*CMT* (pelvic motion tenderness) -*Chandelier sign* "patient presents with generalized pain and just doesn't feel well" Risk factors: -Being promiscuous -having vaginitis *If they have adnexal tenderness (ovaries, fallopian tube) YOU NEED TO US/ADMIT for suspected tuboovarian abscess- usually unilateral*

In a patient with post-partum hemorrhage, develop a medical management plan.

Initial assessment: -Need to determine cause of hemorrhage and attempt to differentiate uterine atony from genital tract lacerations. -Persistent bleeding despite a well-contracted uterus suggests cause is likely from laceration. -If no lower genital tract laceration -> explore uterus for tear -By the time the vitals tank, the patient is in serious condition -Steady bleeding leads to critical conditions -In hypertensive pt. BP will not tank until you are are a BAD state -Must get fluids *quickly*: infuse at 3x the rate of blood loss, use blood and crystalloid solutions *What to give when* NOT ON TEST -Give whole blood if: *severe* hemorrhage, *first line* -Give packed RBCs: if whole blood not available -Give fresh frozen plasma: In DIC with low fibrinogen (no a volume expander) -Give cryotherapy: if patient has volume overload or has a factor deficiency -Give platelets: for low plt count only *Medical Management* -When atony persists despite preventative measures (vigorous fundal massage) use *Methergine injection* (ergot alkaloid derivative) as 2nd line treatment. Increases uterine contractions. May cause severe htn in pre eclampsia. *Only FDA approved method to treat uterine hemorrhage medically.* -Hemabate (carboprost): Form of prostaglandin that causes uterine contractions- also for atony.

Given a patient with PID, choose appropriate medical therapy.

Inpatient Vs outpatient: -will they be compliant? -how bad is the N/V? -is appendicitis still on your ddx? -Are they pregnant? Consider all for determining whether to admit. -Even if the swab is negative, you still treat. Don't wait for a follow up Treatment: *Outpatient*- -Levofloxacin or ofloxacin + metronidazole -Ceftriaxone + Doxycycline +/- metronidazole *Inpatient* -Cefotetan (2nd gen Cephalosporin) + Doxycycline -Clindamycin + Gentamicin

Image Scalp Electrode

Internal electronic fetal monitoring. Scalp electrode penetrates the fetal scalp by means of a coiled electrode.

Given a clinical scenario, correctly choose a contraceptive method from: hormonal, barrier, *intrauterine, post-coital*, or sterilization.

Intrauterine =LARCs, long acting reversible contraceptive. See Below. Post-coital = Emergency contraception = See below

Invasive vs inflammatory breast cancer

Invasive ----------- Two types: -Infiltrating ductal carcinoma -Infiltrating lobular carcinoma inflammatory ------------- -Rare but rapidly spreading cancer -Diagnosed by mammogram and punch biopsy -Treatment via chemo first, then total mastectomy and axillary lymphadenectomy, then chest wall radiation +/- chemo -5 year survival 30-55%

Differentiate between the types of urinary incontinence, include Tx (*stress*, urge, overflow).

Involuntary urinary leakage with increased intra-abdominal pressure such as coughing, sneezing, lifting heavy objects, etc. *leak under pressure* VERY COMMON cause of UI in WOMEN weakness of pelvic floor muscles → often due to multiple deliveries of babies! Tx: Mid urethral slings- Slings are placed surgically that press on the urethra and close it off during pressure. Different types have different placements ○ timed and scheduled voiding ○ kegel's/strengthening ○ topical estrogen, pessaries, collagen injections

*Vaginal repair with mesh* (Sacrospinous suspension)

Just uses a mesh instead of sutures. *does not involve a hysterectomy* Grafts or synthetic Mesh can sometimes cause erosions or pain Mesh is indicated if 1) there is a need to bridge connecive tissue 2) Connective tissue is diseased or weak 3) there is a high risk of recurrence (young age, obeisity)

Determine the *Estimated Date of Confinement (EDC) (due date) using Naegele's Rule* THIS WILL BE ON TEST

KNOW THIS. *To calculate EDC*: 1. *need NORMAL menstrual cycles* 2. *add 7 days to the 1st day of the LMP and count back 3 months and add 1 year* ex: LMP began Sept 10 2017, EDC= June 17 2018 but, gestational (menstrual) age calculated this way assumes pregnancy to have begun 2 weeks before ovulation, which isn't always the case ?? -it's the # of wks since onset of LMP b/c precise knowledge of fetal age is imperative for OB mngmnt and tx options, the appropriate unit= weeks of gestation completed. It is designated using completed weeks AND days, ex 33^4/7 weeks (can also be seen as 33 +4), means 33 completed weeks & 4 days other methods/estimates: -uterine size < small orange ~ 6 wks, large orange ~8 wks, grapefruit ~ 12 wks in 1st tri, crown-rump lengths most accurate tool but the accuracy decreases in later pregnancy

Describe the indications for the long acting reversible contraceptives (LARC).

LARC = birth control that provide effective contraception for an extended period without requiring user action. (IUD, injections, implants) *How it works:* *Release hormone (Progestin) daily through permeable membrane * • Renders endometrium atrophic • Creates thick cervical mucous blocking sperm penetration • Decreases tubal motility • May inhibit ovulation (this is not a guarantee) *Examples:* • Mirena (5-7 years) -Levonorgestrel (progestin) (In photo) • Liletta and Skyla (3 years) -Levonorgestrel (progestin) • Nexplanon (3 years) - Etonogestrel (Progestin) • Copper IUD (10 years) - No hormones *Barriers:* - Cost - Provider doesn't offer *Indications:* - Most effective reversible contraceptives - Rapid return of fertility - No ongoing effort from user is required - Back up contraception: Not required after insertion of copper IUD but Required for others for 7 days after insertion unless placed within 5 days of initiation of menses - acceptable for immediately postpartum, 24% expulsion rate - Failure rate at 1 yr 0.2/100 women (very low, better than tubes tied) *Before you place LARC*: - Routine screening prior to insertion of IUD is not required for low risk women - For high risk patients may screen and place IUD on same day *Side effects (not that common)* • Headache • Nausea • Breast tenderness • Cyst formation • Majority of women still ovulate (63%) *Adverse Effects* : • Infection (very rare) • Bleeding changes (with the LNG-IUS) • Expulsion (less than 5%) • Perforation during insertion or spontaneously (even more rare) • String loss (IUD movement) *Overwhelmingly seems to be a great choice, and if you patient is okay with it, the best option* pregnancy concerns -remove before 14 weeks if possible -abortion rates are high if left in place -ectopic concerns

Summarize the ASCCP recommendations for abnormal PAP smear results: LSIL

LSIL (Low grade Squamous intraepithelial lesion) - Has features of both HPV infection and CIN 1 (Cervical intraepithelial neoplasia) - Much higher chance of progression to CIN 2 or 3 *Colposcopy is indicated* Can also do a co-test with HPV & another in 1 yr

Summarize the complications of untreated PID.

Long term complications: -Infertility (20% risk every time you get PID) -Ectopic pregnancy (due to scarring in the fallopian tube) -Chronic pelvic pain :( Fitz-Hugh-Curtis syndrome is a rare complication of pelvic inflammatory disease (PID) involving liver capsule inflammation leading to the creation of adhesions, that picture is of the liver

Given a clinical scenario, correctly choose a class of steroid contraceptive for a patient requiring it.

Look at contraindications/uses from above/below. • *Estrogen- Progestin combo* o More helpful with PMDD (premenstrual dysphoric disorder) o More helpful with acne o Linked to less menstrual symptoms compared to progestin only o Reduces risk of ovarian and edometrial cancer o Lots of contraindications that would force you to choose progestin only •*Progestin only* o *GOOD FOR* Patients with: - Migraine headache - Over 35 years old and smoker/obese - History of thromboembolic disease - Cardiac disease - Cerebrovascular disease - Early postpartum state - Hypertension with vascular disease over 35 years old - Hypertriglyceridemia

Discuss the management of twin gestation

Management of a twin pregnancy 1st and 2nd trimester: - Early prenatal care - Early ultrasound to determine zygostiy - Scan for anomalies - Cerclage between 16 and 24 weeks if needed (Stitching the cervix shut) 3rd trimester: - Scan for fetal growth (r/o Twin Transfusion Syndrome) - Frequent prenatal appointments limited activity

Summarize the clinical presentation of endometriosis.

May be asymptomatic at first but present with fertility problems May have chronic pelvic pain -Pain is your main clue in -Pain that starts few days before menses (dysmenorrhea) -Pain with defication (dyschezia) -Pain with urination (dysuria) -Pain with sex *(used to never hurt, but all the sudden ouch!)* -noncyclic pelvic pain -The pain is due to inflammation caused by cytokines and prostaglandins released from endometrial implants - could possibly have sciatica or even labored breathing if the tissue is in the lungs. something attacks the nerve and then it suddenly leaves

Summarize the types of first trimester abortion (Missed)

Missed Abortion - may or may not present w/ loss of pregnancy sxs - *closed cervical os* - no POC expelled - *no FHT* - embryo not viable, but retained in uterus -3 management options: 1. Watch and wait 2. Induce labor medically, cervical ripener 3. Suction D&C (dilation and curettage) check blood type if not known, counseling

Understand the pathophysiology of the molar pregnancy.

Molar pregnancy - tumor of placental (trophoblastic) tissue Most common form of molar pregnancy is a complete mole - sperm fertilizes empty egg - Fetus is 46, XX Partial mole is very uncommon - Accompanying fetus is triploid (69 chromosomes) Even less common - placenta remains after previous pregnancy Clinical Presentation: - Persistent bleeding in the early half of pregnancy - Persistent bleeding after a spontaneous abortion or term pregnancy - Uterus large for dates - *No FHT*

If you want to skip your cycles, which type of oral contraceptive do you want to use?

Monophasic

Describe the mechanism for monozygotic twin pregnancy formation.

Monozygotic = One egg splits Placenta and the membranes (chorion and amnion) of pregnancy all depend on the timing of egg cleavage: < 3 days: (2 options) - Separate placentas, Dichorionic, Diamniotic (Separate rooms & beds)(25 - 35%) OR - Fused placenta, Dichorionic, Diamniotic (Separate rooms & beds) 4-7 days: - Fused placenta, Monochorionic, Diamniotic (Same room, separate beds) 8-12 days: - Fused placenta, Monochorionic, Monoamniotic (Same room & beds) > 13 days: Conjoined twins

Compose a treatment plan for a patient with a breast abscess.

Must be drained: either open I&D, or needle via USN

Compose a workup for a patient with nipple discharge.

Nipple discharge is usually benign, but start with a breast exam! Cytology of discharge, if malignant cells are found then it's probs cancer.... but you still have to rule out the cancer if cells aren't found. Get a diagnostic mammogram IF mammogram is negative, get a ductogram for a patient who has consistent bloody or serous nipple discharge. -Most invasive but most definite is subaerolar duct excision (which is also therapeutic)

Discuss the pathophysiology of amenorrhea. (This is really what happens during normal menstrual physiology)

Normal physiology: *Hypothalamus* -GnRH pulses at either high frequency or low frequency *Anterior Pituitary* GnRH at high frequency --> LH GnRH at low frequency --> FSH *Ovary* LH stimulates testosterone production in the Theca cells FSH stimulates aromatization of testosterone in the Granulosa cells In response to LH/FSH the follicles in the ovary produce Estradiol/progesterone Estradiol thickens the endometrium and increases the # of progesterone receptors in the endometrium Progesterone inhibits estradiol's endometrial proliferation effect. As the endometrium thickens and Estradiol/progesterone levels rise this causes negative feedback on GnRH which lowers LH/FSH levels As LH/FSH fall then estradiol/ progesterone fall unless hCG is present (pregnancy) When there is no progesterone continuation (lack of hCG) then you shed the endometrium and have period

(Not specific objective) Post-partum Blues

Not to be confused with post-partum depression. -Shortened period 4-10 days where mom has emotional lability, insomnia -Mood swings are transient -Mood swings are expected and due to wild hormonal fluctuations -Care is supportive

Major risk factors for endometrial cancer

Obesity!!!! HTN TTDM HRT history etc

Discuss the recommended treatment strategy for a patient with uterine cancer.

Official diagnosis and treatment is Laparotomy (Hysterectomy) *Carcinosarcoma Cx you must remove ovaries as well* DO NOT USE HRT in these patients May need Chemo/Radiation as well especially in Stage III/IV disease Stage III/IV -maximum surgery radical hyst with BSO -palliative radiation Worst prognosis: (74% die in 3 years) 1. Leiomyosarcoma (worst) 2. Carcinosarcoma - malignant mixed mullerian tumor MMMT 3. Endometrial stromal tumor (best)

Describe the clinical symptoms for a patient with ovarian cancer.

Often the initial symptoms just appear to be side effects of aging and are not suspicious so it is often missed until late staging. Symptoms: -Increased abdominal size -Bloating -Urinary urgency -Pelvic pain -Fatigue -Indigestion -Inability to eat normally -Constipation -Back pain Some show thrombocytopenia on CBC (>400 platelets) MOST present with palpable abdomino-pelvic mass -*Ascites + Pelvic mass is ovarian Cancer until proven otherwise*

Define infertility as well as subtypes of primary and secondary infertility

Old definition --------------- a. *Infertility:* inability to conceive after 1 year of unprotected intercourse of reasonable frequency. NEW GUIDELINES ------------------------- Less than 35, it is the inability to conceive after 1 year of unprotected intercourse For women > 35, it is 6 months of trying to conceive Over 40 may do immediate evaluation b.*primary infertility:* no prior pregnancies c.*secondary infertility:* infertility following at least one prior conception.

Discuss the pathophysiology of osteopenia and osteoporosis.

Osteo*blasts* BUILD bone, Osteo*clasts* reabsorb bone. *Estrogen inhibits osteoclast activity* Postmenopausal women have decreased estrogen levels so osteoclast activity increases. This, coupled with decreased calcium/vit d and changing PTH levels, leads to: OSTEOPENIA: bone matrix and mineral both decreased, just not as bad as osteoporosis. "Pre-osteoporosis" Then progresses too: OSTEOPOROSIS: Progressively reduced bone mass and strength. This is just how bad the bone is degraded, not due to any one specific pathology, and there are *Two types of osteoporosis:* Primary: due to aging and menopause Secondary: due to disease (Paget's) or medications (chemo) No matter the cause it leads to increased fracture risks: common are vertebrae, femoral neck, wrists (Complications to hip/femoral neck Fx are often fatal in elderly)

Given a patient with *preterm prematurely ruptured membranes (PPROM)*, develop an initial management plan.

PPROM= premature prolonged rupture of membranes (amniotic sac); aka water breaking too early (before 37+0) -it is the cause of 1/3 of premature births RF: -low SEC -low BMI or poor nutrition -smoking -hx of PPROM in prior pre. sx: -PAINLESS rush or steady leakage of fluid/bleeding mngmnt: -*most will deliver w/in 7d of occurrence* -*initiate surfactant at PPROM* to help lungs mature -2 options to manage: expectant or induction -->can deliver right after PPROM occurs or can wait to see if baby gets more fluid reaccumulates UNLESS fever or signs of sepsis. -->if fever or sepsis signs, must deliver NOW. -may or may not do ABX, but incr. risk to fetus concerns: -sepsis or umbilical cord prolapse

Identify the *potential risks* associated with mono-zygotic di-zygotic pregnancies in the *Peri and post-partum periods*.

Peripartum Complications: - Dysfunction al labor - Malpresentation - Abruption (placenta detaches from uterine wall to early, immediate delivery or death) - Abnormal FHR (Fetal Heart Rate) tracing All of the above are cause for a Cesarean delivery Postpartum Complications: - Hemorrhage (Due to uterine atony, (fails to contract after the delivery of the baby, risk of bleeding out)) - Maternal depression (Caring for twin is HARD) - Lactation difficulties - Parenting stress and marital difficulties

Describe the physical findings of a patient with delayed puberty.

Physical findings: *Either no secondary sexual characteristics or growth spurt by age 13, or secondary sexual characteristics but still no period by age 15* Secondary Sexual charachteristics in females: Breasts, wider hips, pubic hair, Etc. *Primary Amenorrhea Definition* -No period by age 13 in the *absence* of growth spurt or secondary sexual characteristics -No period by age 15 in the *presence* of normal growth spurt and secondary sexual characteristics (From Book: Causes: 43% hypergonadotropic hypogonadism, 30% Eugonadism, 27% Low FSH w/o breast development) *Secondary amenorrhea*: no periods after for 3 months after (or oligomenorrhea, which is fewer than 9 cycles/year) (From book, Causes: 67.5% low/normal FSH, 10.5% High FSH level, gonadal failure), 13% high prolactin level, 7% asherman syndome) *These guidelines aren't super strict- for example work-up/testing can be done with patients with stigmata of turner syndrome, obvious virilization, or history of uterine curettage. AND also if patient or parents are concerned.

OCC

Picture is: Ferning pattern can be seen midcycle if cervical mucus is dried on a microscope slide. A high sodium chloride concentration creates a crystalline pattern and is produced by elevated estrogen levels near ovulation.

Review the clinical presentation of a patient with adnexal torsion.

Presentation: -Sudden Sharp Pain that gets worse over hours -Typically will be localized to R side (but can be L) -Radiates to Flank, groin, thigh (If you see the below Symptoms salvage is doubtful because they are signs of necrosis) -N/V -*Fever*

Given a clinical scenario, compose a management plan for a patient with contraindications to hormone therapy.

Pt has vasomotor Sx but has a contraindication *CNS agents* 1) SSRIs- most commonly used Serotonin helps regulate your temperature, these help regulate your seritonin... and they might help with depression Sx. 2)Clonodine- centrally active adrenergic receptor (helps manage Norepi, the other temperature regulator) 3)Gabapentin- no one knows why *Bioidentical hormones* These are *Not FDA approved* They have the same concerns as the previously mentioned medications... so I wouldn't prescribe them... *Comfort measures* Cooling or frozen sheets Freeze underwear Lavender? *Herbs and other witchcraft* Soy: Binds estrogen receptors Dong quai: Exerts estrogen activity... but acts like Coumadin Black cohosh: Most likely exerts estrogen activity (WITCHCRAFT!!!)

*Vaginal repair without mesh* ( Uterosacral suspension or Sacrospinous suspension)

Pulls the vagina into place using sutures attached to a *Ligament* Transvaginal surgical repairs of prolapse that provide vaginal support using the patient's native tissue and sutures *without augmentation by surgical mesh materials* and *frequently involves a vaginal hysterectomy* (if a uterus is present).

Discuss the main causes of a post-operative infection in the C-section patient.

Relatively low occurrence rate (3-5%) -Risk factors include obesity, diabetes, immunodeficiency, pre-existing infection (chorioamnioitis), or stat/crash procedure -Usually a skin bacteria (staph most common) -Occurs underneath (abscess) -This is NOT a seroma/hematoma *Wound dehiscence*: enough infection can separate the facial layer on its own. -One of two things from here: 1) secondary intention closure, 2) return to the OR for intentional secondary closure. -Seromas and hematomas can also cause this.

Describe the *etiology and potential complications of bacterial vaginitis.*

Results from disruption of normal lactobacilli-dominant flora which leads to elevation of pH and overgrowth of anaerobes Common pathogens: SEE PHOTO Common causes: - Menses - Douching - Use of lubricants - Intercourse - Antibiotic Exposure Potential Complications: - PID - Postoperative infection - PPROM - Preterm delivery - Chorioamnionitis - Endometritis

(Practice) Which one is cystocele and which is rectocele

Right is cystocle, Left is a rectocele. If you poke it and it reduces forward its a cystocele, and if it reduces backwards then it is a rectocele.

Discuss the risk factors for developing Uterine cancer.

Risk factors: -Chronic excess estrogen exposure -Tamoxifen use -African American Race -Prior pelvic radiation Protective factors: -COC -Smoking

List the risk factors for adnexal torsion.

Risk factors: -Cyst or mass > 6cm -Being the R ovary -Pregnancy (25% of all torsion cases) -Long uterosacral ligaments being unlucky... (can happen to a normal women with no pathology)

Discuss screening procedures and diagnosis of twin-twin transfusion syndrome.

Screening: Typically becomes clinically evident between 16-26 weeks of gestation so.... - Serial ultrasounds every 2 weeks starting at 16 weeks Diagnosis: *Abnormal Doppler ultrasonography* - Oligohydramnios (deficiency of amniotic fluid) in donor twin with a MVP (Maximal Vertical Pocket) of <2cm - Polyhydramnios (Excessive amniotic fluid) in the receiving twin with a MVP (Maximal Vertical Pocket) of >8cm - *Absent (Empty) bladder in the donor fetus* Treatment: - Laser coagulation of the shared blood vessels can be attempted - Amnioreduction (draining of the excess amniotic fluid on that twins side)

Summarize the types of first trimester abortion (Septic)

Septic Abortion - can occur following any spontaneous abortion - *foul brownish discharge, fevers, chills* - *closed cervical os + cervical motion tenderness* - some POC retained and becomes infected

Image Sinusoidal Pattern

Sine , like on a graphing calculator =(

Develop a medical plan for the patient in #9. (post-operative infection in the C-section patient.)

Single-dose perioperative antimicrobial prophylaxis is recommended for all women undergoing cesarean delivery Treat with antibiotics. May require drainage.

Discuss the workup for an ovarian mass.

Someone comes in with abdominal pain, etc. -Get US of ovaries -Smooth, 1 location, thin wall are all good signs that it is benign -round homogeneous appearing cysts containing low level echoes within ovary suggest Endometrioma -Abnormal findings like teeth/hair are a teratoma... *CA-125 (marker for Ovarian cancer)* Its not a good screening tool, but if suspicious get it checked -Concerning when the level is: >35 in post menopausal women >200 in pre menopausal women -Is the mass nodular or fixed? (both bad) -Do they have ascites or Fm hx of breast or ovarian Cx? -TVUS is frequently chosen as the imaging choice to differentiate benign Vs malignant masses.

Compose a laboratory plan for confirmation of reproductive status and interpret these results.

Someone under 40 comes in and has abnormal bleeding, or thinks they are infertile 2 FSH levels one month apart only if under 40 (FSH high= problem with ovaries, FSH low= problem with the anterior pituitary) This tests to see if the patient has primary ovarian insufficiency (POI) aka premature ovarian failure (POF)which means that the ovaries have stopped working *prior to the age of 40* This *does not include iatrogenic causes* of menopause such as Chemo, radiation, or BSO (bilateral salpingo oopherectomy) If they are over 40 then you don't test, you're just assuming its early menopause.

Summarize the process for staging and determining the prognosis of cervical cancer.

Staging of Cervical Cancer: FIGO System - Done Clinically - *Does not include TMNO system* - *Does not include lymph nodes* - DOES include size of lesion - DOES include spread to distant organs 2 MAIN TYPES OF CERVICAL CANCER 1. *Squamous Cell 70% of cervical cancers* - Declining in recent years due to HPV screening - 2 subtypes (keratizing and non-keratizing) 2. Adenocarcinoma *Usually advanced before becoming evident* - Give the cervix a *barrel shape* - 7 subtypes - Overall worse prognosis

Determine screening recommendations for osteopenia/osteoporosis

Started at *age 65* or with an indicating fracture (any non traumatic fracture) Gold Standard for testing = *DEXA* Dual Energy X-Ray Absorptiometry Tests *lumbar vertebrae, radius, and femoral neck*

Paget's disease: not sure we need to know this

Subset of DCIS -Ductal cells that migrate to the surface cause an eczematous rash -Can palpate mass a majority of the time and is usually seen on mammogram -Definitive diagnosis is a punch biopsy of the affected nipple -Treatment is wide excision with clear margins ¡Follow with radiation

Describe the staging system for a patient with ovarian cancer.

Surgical staging is required FIGO staging is how this is staged *Stage I*-Tumor confined to ovaries (or to fallopian tubes *Stage II*-Tumor involves 1 or both ovaries (or 1 or both tubes)a with pelvic extension (below the pelvic brim) or primary peritoneal cancer *Stage III*-Tumor involves 1 or both ovaries (or 1 or both tubes)a with confirmed spread to the peritoneum outside the pelvis and/or metastasis to retroperitoneal lymph nodes *Stage IV*-Distant metastasis excluding peritoneal metastasis

Treatment and Follow-up Guidelines for Cervical Cancer (maybe an objective?)

TREATMENT - Stage 1B1 or lower --> radical trachelectomy (remove cervix/ lower uterus) and pelvic lymphadenectomy if desiring fertility (can still carry baby if IVF) - Starting at IIB it is chemoradiation - IVB: palliative chemo/radiation/hospice FOLLOW-UP *Every 3 months for 2 years* Then... *Every 6 months for 5 years* Then... Annually afterwards Every visit: - Pelvic - Complete Node Survey - Pap test

Given a clinical scenario, correctly choose a contraceptive method from: hormonal, barrier, intrauterine, post-coital, or *sterilization.*

TUBAL INTERRUPTION 1. Electrocoagulation (highest serious complication rate) - Where you cauterize and cut on through 2. Mechanical Methods (clips, rings) - filshie clips, important to look for blanching of the tubes or else you messed up 3. Suture Ligation (favored during C/S with BTL (cesarean section with bilateral tubal ligation) - Old method, 2 double knots on the end of the tubes. Must be done correctly. *Counseling:* permanent vs reversible (things with rings and clips are more reversible, but nothing is guaranteed) - No effects on menstrual control TRANSCERVICAL • Reaches tubal ostia and causes an occlusion via mechanical or chemical • NOT IMMEDIATE and requires FOLLOW UP TESTING 1. ESSURE (FDA approved) - coil into each ovarian tubes (See photo), want scars to form, tubes stay forever. Check scar tissue by injecting dye and making sure it doesn't reach the ovary, 100% by 6 months if placed correctly 2. Pellet placement (WHO recommends against this-carcinogenic possibilities) *Male Sterilization:* Vasectomy - 0.15% failure rate at 1 year - Not immediately effective • Require alternative contraception until azoospermia • Confirm with 2 consecutive semen analyses BETTER THAN FEMALE STERILIZATION

Describe the pathophysiology of a *Bartholin's Gland Duct abscess*.

The Bartholins Glands are located at "4 o-clock" and "8 o-clock" of the vaginal opening. They secrete mucus to lubricate the vagina. When the glands become obstructed, they can develop into a cyst. Once that cyst becomes infected, it is an abscess. *Most common pathogens* - Gonorrhea - Chlamydia - Coliforms - Anaerobes

Summarize the physical symptomatology that accompanies the physiologic changes listed in #2 above. (perimenopause, menopause, and post-menopause.) *overview card*

The age and stage at which these changes occur vary from woman to woman *AUB* abnormal uterine bleeding *vasomotor Sx* AKA hotflashes *Weight gain* *Skin changes* *Dental changes* *Sleep changes* *high risk of depression* *Lower reproductive tract changes* *Changes in sex* *Cardiovascular changes* *Changes in bone mineral density* If you need a refresher on any of these see the next like...9 cards for more detail

Discuss the pathophysiology of abnormal uterine bleeding (AUB).

The endometrium is what bleeds 2 layers: -Functionalis -Basalis The main player is *functionalis layer* which is responsive to hormones -This layer breaks down and sloughs off (bleeding) -When you have abnormal bleeding there is often a problem with artery constriction not working properly. Due to hormone imbalance. AUB is defined as bleeding > than 7 days or more than 80 mL of blood per cycle anovulation is usually the cause of AUB, but dont forget palm coein

Summarize the expected normal course of recovery for a non-complicated C-section.

The normal post-partum course for *C/Section* -Physiologically speaking, does not vary much from vaginal delivery as cervix is generally opened during operative delivery. -Blood loss is generally greater (1500 is avg) -Return to bowel functions generally 8 hours -Return to bladder function generally 12 hours -Hospital discharge on 3rd or 4th post partum day

Summarize the expected normal course of recovery for a non-complicated vaginal delivery.

The normal post-partum course for *Vaginal Delivery* -Uterine vessels begin to return to normal size -After-birth pain marks the uterine contractions that may persist for several weeks (also occur with breast feeding) -Myometrium involutes to decrease uterine size -Decidual tissue sloughs off: lochia --Lochia rubra (red) x 3-5 days --Lochia serosa (pink/brown) up to 10 days --Lochia alba (white/yellow) until 24-36 days --Lochia should steadily decrease in color

Outline a treatment algorithm for a patient with endometriosis.

The treatment varies widely depending on how bad the pain is and whether the patient wants to become pregnant at some point. *If pain is the problem:* -NSAIDs and OCP(depoprovera: bone density loss risk ~5 yrs) -LARC's (Long-acting reversible contraceptives): works unless in bowel tissue -GnRH agonist (Lupron) puts the Pt into pseudo-menopause. (need add back therapy: give them just enough estrogen to prevent the menopause side effects. -Aromatase inhibitors (letrozole/anastrozole): could increase ovarian cysts (add progestin) -Laparoscopic resection of endometrial tissue -Hysterectomy w/wo oophorectomy: leaving the ovaries may cause the pain to persist. Take em out

Summarize the types of first trimester abortion (Threatened)

Threatened Abortion - bloody vaginal discharge - *closed cervical os* (don't check cervix after first trimester, you have to get US first) - during first 20 weeks. -US: you see heart tones.... the only thing you can do is get blood type. - 50% will proceed to abort - *FHT still present* -Counsel the patient on what may happen and when to come back

Summarize twin-twin transfusion syndrome.

Twin-twin transfusion syndrome (TTTS) can occur in pregnancies when twins share a placenta. Abnormal blood vessel connections form in the placenta and allow blood to flow unevenly between the babies. - Occurs in 10-20% of monochorionic pregnancies (most important cause of mortality) Possible Outcomes: - Hydrops Fetalis (serious fetal condition defined as abnormal accumulation of fluid in 2 or more fetal compartments, including ascites, pleural effusion, pericardial effusion, and skin edema) - Death of one or both twins

Given results of a bone mineral density test, be able to interpret results.

Two Scores: *T and Z* *T Score:* Compares patient to person of same sex at *peak bone mass*: this is the one you use to diagnose osteoperosis: Works in Standard Deviations A score of -2.0 means the patient is 2 SDs below the peak *Normal is +2.5 to -1.0* *Osteopenia -1.0 to -2.5* *Osteoporosis at or below -2.5* *Severe is -2.5 PLUS 1 or more fractures* *Z score:* Compares patient to average bone mass for patient with *same age/weight* Also uses Standard Deviation If a patient falls at -2.0 on a Z score, they are below their age range and you should *workup for a secondary cause*: (Thyroid,Renal,cancer,Liver,Alcohol) Look at Z score for someone a little older, T score averages scores from young people.

Given a patient with adnexal torsion, construct an appropriate treatment plan.

Tx: Laparscopic Surgery -Try to salvage the ovary -Don't remove the cyst when its inflamed (remove later) (use OCP for 6-8 weeks to control cyst) -Wait 8-10 min in OR to see it pink up -Wait 24-48 hrs to check for reprofusion in cases where it initially doesn't pink up and no signs of necrosis - if cyst is hemorrhagic and big, drain it! then take it out. poke a hole in it yo -If the torsion is necrotic then remove all together. In a *pregnant patient* if you remove the corpus luteum cyst during 1st trimester you must *supplement progesterone until 10 weeks gestation* you need progesterone to support a pregnancy until week 12.

Given a clinical scenario, construct the appropriate imaging to confirm a patient has an adnexal torsion.

ULTRASOUND!!!! -Look for ovarian congestion/edema *"Bulls eye target"* whirlpool, snail shell -Doppler US will show the interrupted blood flow ( you're only allowed to be suspicious with US) -CT only used in a complicated case Diagnosis is on VISUALIZATION in OR with laparoscopy

Normal ovary vs endometrioma on US

US: you can see endometriomas at 20 mm or greater, ground class appearance CT may show endometriomas as well as extra-pelvic implants

*Abdominal repair with mesh* (Sacrocolpopexy)

Use mesh to fix the vagina in place against the sacrum More erosion, and pain, *but the pt can be more active* and this surgery returns normal vaginal anatomy Abdominal surgical repair of prolapse involve minimally-invasive techniques (such as laparoscopic or robotic instruments) to support *the pelvic organs using surgical mesh* and frequently involves a hysterectomy (if a uterus is present).

Describe natural family planning.

Using a calendar and measuring your body temperature (rise in temp of 0.5 F to 1 F), cervical mucus (thin, "stretchy" clear cervical mucus), and days between cycles to determine when you are most, and least fertile. *Most fertile from days 9-19 of your cycle.* For max success, requires: - proper education - people who actually will keep track - people who will actually abstain during fertile days (or willing to use barrier method in addition) - *REGULAR CYCLES.* - someone who is willing to wear this stylish bracelet... so trendy.

Compose a medical treatment plan for pelvic organ prolapse

What you do *depends entirely on how bad the pt's Sx are.* *observation*- if it isn't a problem for the pt. You don't have to do anything. *Pessary*- They feel a slight bulge, or are having minor urinary problems, but not enough to justify surgery. (remove nightly to weekly and wash)- or if they have digitation. For patients who are not a good surgical patient. *PFPT*-( Pelvic floor physical therapy)-strengthening pelvic floor can improve Sx. No matter what other treatment you choose. This Improves muscle strength and coordination, but will not change anatomy if Sx are severe. *surgery*- will alleviate Sx and some will preserve sexual function

Discuss the physiologic changes that occur in perimenopause, menopause, and post-menopause.

When these changes occur varies greatly from woman to woman *perimenopause/menopause* *Hypothalamus-Pituitary-Ovarian Axis* -FSH levels rise slightly (as your follicles produce less inhibin) in early MT giving increased estradiol -As MT goes on, free/unbound estrogen/testosterone increases -Estradiol does not drop until late MT *Ovaries* -Ovarian senescence (egg loss) begins in utero and progresses (rapid in 30s and 40s) *Adrenal Steroid Levels* -DHEAS (adrenal steroid that is a precursur to sex hormones) declines after age 30 *Endometrium* This has big changes depending on what stage you are in. -Early MT endometrium follows ovulatory cycles like normal. -Late MT *estrogenic effect* there is no progesterone opposition (occurs when there is no ovulation) and leads to buildup of endometrium -Post menopause: atrophy due to lack of estrogen stimulation

Compose a counselling plan regarding theoretical failure rates of the methods listed in #3 above. (hormonal, barrier, intrauterine, post-coital, or sterilization.)

Which method is right? Have a conversation with your patient. o Lifestyle, Partner choice, Desire for fertility, Long term outlooks, Co-morbid conditions, Patient Research - Educate the patient on all options - Discuss their personal best options - Use MEC criteria to guide patient in best options (See final cards for examples) - Be honest about adverse effects and failure rates *Use the CDC Medical Eligibility Criteria, or MEC*. o Useful tool to determine from 1-4 how safe a method is for your patient (see next card picture)

OCC Drugs that effect Birth Control effectiveness

Will need back up contraception method if taking one of these.

Given a clinical scenario, construct an appropriate initial workup for a patient with suspected PCOS.

You should have clinical suspicion based on: (Rotterdam Criteria) 1. Abnormal uterine bleeding pattern 2. Evidence of hyperandrogenism (hair, acne, enlarged clitoris, decreased breast, increased muscle mass) 3. US show more than 12 small cysts on ovaries Labs: (many labs are run to rule out other causes) *LH*-Elevated *FSH*- usually lower by a 2:1 ratio (would be elevated in early menopause) *Free Testosterone Level*-Elevated *DHEA-S*- If this is elevated it means that the androgens are coming from the adrenal glands (Problem isn't PCOS) *Prolactin*- Elevated would suggest prolactinoma (Problem isn't PCOS) *TSH*-Rules out hypothyroidism as the cause of the amenorrhea ((Problem isn't PCOS) *Insulin:glucose ratio*- If they have high levels of insulin that supports the diagnosis of PCOS

Describe the etiology of each of the Pelvic Organ Prolapses (POP) (cystocele, rectocele, and uterine prolapse)

Your pelvic organs (*uetrus, bladder, bowel*) sit on your pelvic floor muscles, and are attached to the pelvis by ligaments and fascia. When the pelvic floor is normal there is no ligament tension. When you have a trauma, a baby, or a disease that weakens this area your pelvic floor is stretched and sags. If the muscles are not quickly strengthened back to normal for support, the organs put pressure on the *ligaments*, which are stretched as well over time. Eventually the pelvic floor and ligaments become so weak/stretched that the organs prolapse down The main muscle you are worried about being directly damaged, or suffering nerve damage in the pelvic floor is the *levetor Ani* Sx:Pretty wide range Pelvic pressure Pelvic pain Feeling of a "lump" or a vaginal "bulge" Back pain Urinary dysfunction Bowel dysfunction Water= pelvic floor Boat= pelvic organs Ropes= ligaments

Review Leopold's maneuvers to determine the position of the fetus.

a. *Abdominal Palpation Method* - really experienced people can determine if baby will be breech and size of baby b. Almost impossible to perform in situations of obesity, polyhydramnios, or placental abnormalities i. *1st* Start by determining *which part is in the fundus*, butt is large nodular mass, head is hard mobile mass. Now determine fetal lie. ii. *2nd* Then palms on either side-is there a hard back? *Which sides are the back and limbs facing*? iii. *3rd* step is determining *which part is in lower pelvis*-should be opposite of the fundus iv. *4th* and finally, turn toward mom's feet and try to *confirm head or anterior shoulders in the pelvis*

Describe the medical therapies available for correction of male infertility.

a. *Aspermia = no semen from failure to ejaculate* i. Anejaculation/anorgasmia: TRX: varies based off cause, counseling, Viagra, vibratory stimulation. For spinal cord injuries can use electroejaculation... just google it if you want to know more. ii. Retrograde ejaculation: Patient has orgasm with no or very little ejaculate because sperm goes into bladder. TRX: alpha adrenergic agent, collect sperm from urine iii. Hypospermia = <2mL, TRX is same as all above iv. Duct Obstruction: transurethral excision of the obstruction and reanastomosis of the ejaculatory ducts b. *Abnormal Sperm Count:* Male produces semen but there aren't a lot of sperm in it. i. Azoospermia: no sperm in semen - Obstructive Causes: prior vasectomy, congenital absence. TRX (to obtain sperm, likely for IVF): surgery if possible or testicular sperm extraction (TESE) or intracytoplasmic sperm injection (ICSI, put sperm in egg under microscope see photo) - Nonobstructive causes: genetic, testicular failure, unknown causes c. *Environmental influence* i. Make sure testicles aren't too warm or beaten up. d. *Abnormal Sperm Motility or Morphology* i. Asthenospermia: decreased sperm motility (they swim in circles). TRX: testicular sperm extraction (TESE) or intracytoplasmic sperm injection (ICSI, put sperm in egg under microscope) ii. Teratozoospermia: Abnormal sperm morphology. TRX: try ICSI, but don't get your hopes up. e. *Infertility from varicocele* i. TRX: Nothing proven to improve fertility

Given a patient in labor, explain the indications for use of operative delivery (aka assisted vaginal delivery).

a. *Defined by the use of vacuum device or forceps* b. Operative delivery occurs ~4% of births c. Generally successful in giving vaginal births d. *Done in order to relieve a condition that will have a negative outcome on the mother or fetus that will otherwise improve with delivery* *Indications*: (1) Nonreassuring fetal heart tracing (2) Premature placental separation *Maternal indications* (a) Heart disease (b) Pulmonary injury/compromise (c) Intrapartum infection (d) Exhaustion (e) Prolonged second stage labor -Over 3 hours with or over 2 hours without epidural in nulliparous women -Over 2 hours with or over 1 hour without epidural in multiparous women

Discuss indications for In Vitro Fertilization.

a. *Mature oocytes* from stimulated ovaries are retrieved transvaginally with sonographic guidance. *Sperm and ovas combined* in lab to make *several embryos*. Best viable options are *transferred* transcervically into the *endometrial cavity*. b. Recommended if attempts at other medical options have failed, or unable to obtain proper sperm sample. c. Most effective method, bypasses several possible barriers to fertility d. *Very expensive* This is what you would do if you had a kid with duchennes and didn't want to go through losing a kid again, they can choose the outcome pretty much

Describe the initial laboratory evaluation for a female infertile patient.

a. *Menstrual history:* if menses 24-25 days - 95% ovulatory, but if irregular or >35days then oligoovulatory b. *Ovulation tracking:* i. If progesterone is < 3 on day 21 of cycle, then patient didn't ovulate ii. No mid cycle basal temperature rise c. *Labs to get* 1. Galactorrhea- Prolactin 2. Thyroid sx- TSH 3. Androgen excess - Testosterone, DHEAS, and 17-OH- progesterone 4. PCOS - HgA1C- really common 5. LH &FSH (If FSH over 10, indicates ovarian reserve loss (POF)...early menopause) 6. Consider serum progesterone in the luteal phase d. *Tubal Factors* (If no diagnosis from labs, proceed to this) i. Risks: PID, STDs, surgery, ectopics ii. Diagnosis: HSG in follicular phase (Hysterosalpingogram), sonohysterogram, hysteroscopy, laparoscopy e. *Unexplained couples: 15-20%*

Summarize the initial physical examination needs for a male patient with infertility.

a. *Questions to ask:* i. See female questions and pick the ones that work for guys. b. *Physical Examination* i. Normal signs of testosterone production? = secondary sexual characteristics, beard growth, axillary, pubic hair, maybe male pattern baldness ii. Gynecomastia? Maybe Klinefelter syndrome iii. *Genital Exam* - Urethra at tip of glans? - Testicles at least 4cm, volume of 20mL - Masses? - Epididymis soft? Nontender? (tender = chronic infection, fullness feeling = vas deferens obstruction) - Prostate? Smooth, nontender, and normal size? - Pampiniform plexus of veins normal size? ( varicocele) - Vas deferens present? (bilateral absence seen with cystic fibrosis)

Describe the initial laboratory evaluation for a male infertile patient.

a. *Semen analysis:* Obtain after 48 hours of abstinence and deliver in 30 minutes i. Normal Results: ii. Volume: 2-5ml iii. Concentration: >20 million/ml iv. Motility: >50% v. Strict morphology: >12% normal vi. Sperm antibodies vii. Appropriate amount of White blood cells

Summarize the lifestyle therapies available for an infertile couple.

a. *Weight Optimization* (1st line for obese infertile) i. Too low = associated w/ hypothalamic hypogonadism - Anorexia and Bulimia can cause undernutrition which is closely linked to the reproductive axis ii. Too high = central body fat assoc w/ insulin resistance, PCOS. Loosing weight can decrease hyperandrogenemia, lower LH concentration, and resore normal fertilization (even 5-10%) - Ideally additional treatment postponed until under BMI 40. b. *Exercise* i. Relationship to fertility is not clear ii. But if working out too much causes low body weight, then adjust, and if no exercise leads to high body weight, then adjust. c. *Nutrition* i. Female: 400mcgs folic acid daily ii. Male: Theory= antioxidants help sperm DNA, iii. Both: Nutritional supplements, diet modification, daily multivitamin d. *Stress Management* i. Female: severe stress can cause anovulation ii. Patients with higher stress levels have lower IVF success (unclear why), medication not recommended, but CBT and meditation may help.

Describe the etiology of infertility.

a. Affects 10 to 15 percent of reproductive-aged couples b. For a successful pregnancy need the following to all work correctly together: i. Ovulation (most common problem), ovum pick-up by a fallopian tube, fertilization, transport of a fertilized ovum into the uterus, sperm of adequate number and quality, and implantation into a receptive uterine cavity. ii. If a couple is infertile, you are trying to figure out which of these steps is going wrong. c. In general; the fault belongs to the *female 1/3, male 1/3, and the couple 1/3.* d. With females, possible physical problems: i. Diminished ovarian reserve due to aging ii. Anatomical abnormality iii. Tubal factors; proximal or distal iv. Uterine factors: Leiomyomas, endometrial polyps, intrauterine adhesions v. Peritoneal factors; Endometriosis, pelvic adhesions vi. Cervical factors e. *MYTHS:* Things that have no effect on fertilization: i. Having sex too much, while daily sex may decrease sperm count, it's not significant enough to mean anything ii. Sex position iii. Remaining horizontal

Discuss indications for Intrauterine Insemination.

a. Good normal, *motile spermatozoa* are separated from dead sperm and other byproducts then via thin flexible *catheter, placed in the uterine cavity*. Supervision not needed. Try to time this with the LH surge. b. Recommended for treatment of cervical factors, mild and moderate *male factors and unexplained infertility*

Construct a plan to maintain child bearing options for a patient faced with chemotherapeutic induced loss of fertility.

a. Have this conversation with all of your potential cancer patients. They have the right to be told to consider these things. Not tell them is taking away their choice. *Fertility Preservation (FP)* *FEMALE:* - *Prepubertal:* a. Immature oocyte cryopreservation (freeze egg) b. Ovarian transposition (moving ovaries out of the way of radiation) - *Post-pubertal:* a. Oocyte cryopreservation (freeze egg) - banking, harvest and freeze unfertilized eggs, best in women under 38. Concern for damage of lipids when freezing. No partner needed. b. Embryo cryopreservation (freeze embryo)- banking, if patient is well enough. Mature eggs removed, fertilized, frozen and stored. Need 2-3 weeks for process. This option used if there is an established partner. e. Ovarian transposition (move the ovary out of the radiation beam path) - *After Cancer* a. Should wait to conceive for 6 months b. No difference in birth defect rates b/t those with history of cancer and those without *MALE:* -*Prepubertal: * a. Testicular Tissue cryopreservation (freeze sperm) b. Shiedling testis from radiation -*Post-pubertal* a. Semen - cryopreservation and banking, ~$1000, sperm frozen after masturbation, sperm frozen indefinitely, use in future with IUI or IVF -*After cancer* a. Should wait 1-2 years before trying to conceive b. No difference in birth defect rates b/t cancer survivors and people with no cancer history

Understand the importance of the fetal heart tracing (FHT).

a. Previously just used with high risk pregnancies b. Now used frequently (over 2/3rd of all pregnancies) c. Normal to drop temporarily after each pushing session d. Can be direct or indirect - Direct is via a scalp electrode (requires ruptured membranes) - Direct is considered a strong indicator - Indirect (via two bands over mom's abdomen) - Somewhat more controversial as there is not a worldwide standard *Baseline Activity:* -*Rate (normal is 111-160)* -Beat to beat variability (important index of function, regulated by autonomic nervous system) -Should have *3-5 cycles* a minute -Persistently flat FHT may indicated nervous system damage a) Arrhythmia (difficult to interpret on FHT alone) b) Sinusoidal pattern (if not due to narcotics, likely bad news): Smooth undulating sine wave pattern. May be terminal rhythm... =( *FHT accelerations and decelerations:* *Accelerations* (1) Should occur with fetal movement, fetal stimulation, labor (2) Reassuring for denying presence of acidemia *Decelerations* (1) Early: comes on gradually and returns to baseline, occurs with contraction (2) Late: can either be due to contractions (late labor) or placental function - Smooth decrease in FHT at the contraction peak, returns to baseline only after the contraction has ended - Late decels will also end up in C/S *Variable Decelerations:* abrupt decels lasting less than 30 sec

What is chronic PID associated with?

actinomycosis (more likely with IUD- don't pull it out in acute setting! ) tuberculosis

Discuss *resolution of discrepancies between ultrasound dates and uterine size dates*

answered above

ductal carcinoma in situ: not sure we need to know this

breast cancer at its earliest stage before the cancer has broken through the wall of the milk duct -Cancer develops in the mammary ducts without spread beyond membrane -Cannot exist outside of the duct (Stage 0 breast cancer) -25-30% of breast cancers in the US -Can be seen on mammogram and may have palpable mass -Treatment is excision with clear margins (may require mastectomy) -Radiation to breast after surgery improves outcomes to 96% -5 years of Tamoxifen to prevent reduce recurrence or contralateral cancer

Compose a surgical treatment plan for each of the defects listed above.

can be *reconstructive* or *obliterative* *Reconstructive*- (pexies, rraphis) Attempts to *re-create functioning pelvic organs*, *Maintains ability to have intercourse*, May use augmentation materials, like mesh Can be posterior, anterior, or lateral colporrphay depending on what part of the vault is prolapsed down. Ex) Vaginal repair without mesh Vaginal repair with mesh Abdominal repair with mesh *Obliterative* (Colpoleisis) Easier surgery with better success, but they *CANT HAVE SEX* Narrows the vaginal opening Doesn't involve use of mesh for prolapse repair Minimizes surgical risk, with excellent durability

Discuss screening for *HPV* i pregnancy

condyloma acuminata- will almost always increase in pregnancy if pt has hx -if they occlude vaginal opening, may need C/S if acid solution doesn't work to clear them. do not try laser ablation b/c can scar. -biggest risk to fetus: neonatal infection respiratory papillomatosis (benign larynx neoplasm) that can cause RDS.

What to assess during prenatal visits (not an objective)

fetus: fetal HR, growth, amniotic fluid volume, fetal activity mother: BP, weight, extent of change (uterine exam measure size from symphysis to fundus), symptoms: H/A, vision changes, abd pain, N/V, bleeding, vaginal fluid leakage, dysuria -estimate pelvic capacity & its configuration, amniotic volume adequacy, cervical consistency, effacement, & dilation -height of uterine fundus (from top of symphysis pubis to top of funds) btwn 20-34 wks correlates closely w/ gestational age & is used to monitor fetal growth & amniotic fluid volume; limited by obesity or uterine masses- get US for these cases -Doppler US: can detect fetal heart sounds by 10 wks in absence of maternal obesity (normal fetal HR: 110-160 bpm)

Discuss screening for *HSV* in pregnancy

genital herpes: either HSV1 or HSV2 -transmission: intrauterine, permpartum, or postnatal via ROM or through lesion contact at delivery -transmission likeliness increases w/ operative delivery and timing of HSV infxn *(recent infxn at time of delivery is highest risk up to 50%, i.e. 3rd tri herpes=bad)* newborn infxn: can be localized or disseminated (multiple organs) -fatality 50% in encephalitis or dissemination screening: *NOT screened for routinely* -only screen if they have symptoms via viral culture of lesion w/in 24 hours and back-up serology tx: primary outbreak: antiviral therapy for outbreak PLUS suppressive therapy for remainder known HSV infxn: initiate suppressive therapy at 36 wks (in addition, tx if they have new outbreak) any lesion in vaginal area (vulva, vagina, or cervix) OR any prodromal sxs'--> C/S -->no genital lesions, can do vaginal delivery

Summarize the *ToRCH panel.*

group of blood tests to test for infections that can cross placenta & infect newborn *Toxoplasmosis*- transmitted via cat feces -make husband scoop the poop :) don't let him say no but if he does, wear mask/gloves & scoop w/in 24 hours *Other (Syphilis)* *Rubella*- prevent via immunity *Cytomegalovirus*- college students/day care (no tx) *Herpes Virus*: herpes type 2 matters more sx: -generally aren't present until birth but may see some on US dx: -Ab titers if suspected exposure in pregnancy tx: -diagnose promptly and plan delivery accordingly (i.e. not in BFE) sequelae: -hydrocephalus -cardiac malformations -microcephalus -hypoplastic limbs (underdeveloped limbs) -permanent retinopathy

Summarize the appropriate *routine laboratory testing to be done at the first trimester, second trimester, and third trimester*

http://accessmedicine.mhmedical.com/ViewLarge.aspx?figid=59790717&gbosContainerID=0&gbosid=0 *1st prenatal encounter:* CBC- anemic? Urinalysis & Urine cx (r/o asymp. bacteria) PAP if not UTD CF (cystic fibrosis) Fetal Aneuploidy- normal 46 comes or trisomy 21 Rh & Ab status to determine blood type Rubella immunity HIV HBV syphilis, CT (add gonorrhea if high risk) *15-20 wks (2nd tri):* repeat fetal aneuploidy open neural tube defects (spina bifida) CF if wasn't done in 1st visit *24-28 wks* CBC Glucose tolerance (most sensitive wk 26) repeat Ab screen if found RH- & give anti RH IG if still unsensitized *29-41 wks* Fetal Aneuploidy repeat GBS (group B Strep) via vaginal & rectal cultures; if positive- ABX --->nml vaginal flora but can cause fetal meningitis upon delivery & IF HIGH RISK ( mult. sex partners, <25 y/o, hx of STDs, little prenatal care, IV drug user)- repeat syphilis, GC/CT, HIV, HBV

Describe shoulder dystocia.

i. *Obstetrical EMERGENCY* ii. Body becomes trapped after delivery of head iii. *Anterior fetal shoulder trapped behind pubic symphysis* - This is bad news because the cord is now trapped w/in birth canal iv. Teams practice a shoulder dystocia delivery due to emergency status v. Risk ranges from *severe neuromusculoskeletal injury to death for fetus* vi. Mom may have hemorrhage to significant lacerations *Risk Factors:* (i) Macrosomia (ii) Over 75% were fetuses over 4000 g (iii) Prolonged 2nd stage labor (iv) Operative vaginal delivery (v) Prior shoulder dystocia

Most common ectopic pregnancy implantation site is? TEST QUESTION: cough cough

in the tubes man! Ampulla: fallopian tube

Given a mother with *recurrent UTIs*, describe the expected maternal and fetal complications.

main cause= asymptomatic bacteriuria -this is why you do urine cx at 1st prenatal visit (and bc pregnant ppl pee a lot so easily miss confuse frequency w/ being pregnant) RF: -DM -sickle cell trait pts ^^ *these pts. must get urine cx EVERY MONTH* dx: -greater than 100K organisms/mL clean catch tx: -*Nitrofurantoin (Macrobid)* x10d for initial episode. -2nd episode: x21d -3rd episode: tx until delivery -*test of cure: i.e. have them F/U and re-test to make sure it is gone* sequelae: -if UTIs go untreated: low-birthweight infants for fetus, anemia and GHTN for mom

Given a patient in *preterm labor with intact membranes*, develop an initial management plan

mngmnt: 1. *bethamethasone*- single dose for fetal lung develop (instead of surfactant as in PPROM) 2. *Tocolytics* ( Terbutaline or Nifedipine)- to delay labor 3. +/- *Mag Sulfate*- i.e. if large contractions are on monitor 4. +/- *Cerclage*- cervical stitch 5. +/- Bedrest *Tocolytics- SHORT TERM ONLY* since can cause pulmonary edema or maternal death. -->*do NOT use if in labor aka only use if no evidence of cervical change* conclusions: you can't stop labor if it wants to happen (t's a train on a track w/ a final destination). the goal in PTL: give lungs as much chance to develop as possible

Discuss *testing for ZIKA virus*.

mosquito-borne virus; can be also transmitted via sex *screening: 2 options* 1. serum RNA (not invasive-needle stick) or 2. nucleic acid amplification of amniotic fluid (very invasive) -also serial US to evaluate baby -can check neonate serum & urine via PCR or CSF sequelae/effects: -IUGR (intrauterine growth restriction) -microcephaly -ventriculomegaly

Summarize the *specific tests recommended for high-risk pregnancies* (specifically those related to *mothers with hypertension, diabetes or a history or gestational hypertension or diabetes*).-

need detailed hx of prior pregnancies & assoc. conditions (gestational DM, gestational induced HTN) since many OB complixns tend to reoccur in normal pregnancy early on, BP lowers d/t incr. progesterone. later in the pregnancy, BP increases d/t decrease in progesterone. *Pre-Existing HTN*= either come in with hypertension at the 1st visit OR if it's HTN diagnosed *BEFORE 20 weeks gestation is completed* (i.e. 19 + 7 counts as pre-existing HTN) -incr risk for: CVA, kidney damage, aortic dissection, placental abruption (d/t incr. in placental vessels), growth retardation, superimposed preeclampsia tx ASAP: *-Adrenergic blockers (labetalol, methyldopa): most commonly used & very safe* -Calcium Channel Blockers (nifedipine-but not for severe gestational HTN) -Vasodilators (hydralazine): only a RESCUE drug -Thiazide Diuretics- controversial; many AE's *never use ACEi (teratogenic)* *Preeclampsia* -order: 24 hour urine (look for PRO), CrCl, CMP (liver & kidneys), PT/INR -pts w/ preexisting HTN are at incr. risk -monitor w/ US (??): asses fetal growth, placenta sufficiency *Mom w/ hx of Gestational HTN*: tx as chronic HTN -they do not have HTN or current gestational HTN -get baseline labs (on all prenatal visits) -q2 wk apts. as opposed to q month *Diabetes* -incr rates of Spontaneous Abortion, preterm delivery, fetal malformations (caudal regression aka mermaid syndrome), & macrosomia (big baby), shoulder dystocia, C-section -infants are at incr risk for: Respir. Distress Syndrome (RDS), hypoglycemia, hypocalcemia, long term brain development -*DM is affects baby more than the mom* -*AFP or Quad screening AND a level 2 US in 2nd tri for all DM pts.* d/t risk of neural tube defects -*incr. US frequency in late pregnancy* -tx: TIGHT CONTROL OF BS *insulin= 1st line in pregnancy* -don't just deliver baby based on size (big does not mean mature) *mom w/ hx of Gestational DM* -early testing of glucose tolerance test in wks 11 to 14 after you get rapid BS to r/o elevation. If tolerance test neg, repeat in wks 24-48 AND get BS on mom (any method) 6 wks post partum to ensure it resolves (if positive-->PCP)

Summarize the *genetic testing* recommended for specific age and ethnic groups.

offer *PRECONCEPTION* screening for certain genetic abnormalities to increased risk pts based on fam hx, ethnicity, *advanced maternal age (AMA= 35 y/o AT TIME OF DELIVERY)* -none of these tests are guaranteed, even if positive *Tay-Sachs disease:* Eastern European Jews or French Canadian, Cajuns ->enzyme deficiency dz, premature death ~5 y/o and is indication for medical termination ->preconception: screen partner who is descendent. if already pregnant, screen both ->*if positive, do invasive testing: amniocentesis or chorionic villus sampling (CVS)*; chorion= a fetal membrane that attaches to uterus *Sickle Cell & Thalessemias:* African, AA, Middle Eastern, Asian Indians, Mediterranean, Alpha-Thalassemia: SE Asian, African -->*order Hb Electrophoresis*: blood test that detects HbS *Trisomy 21 (Down Syndrome):* AMA (>35) AND adolescents *1st tri DS screening (do both 1&2 btwn wks 11-14)* 1. US to measure nuchal fold 2. Serum Analytes of pregnancy associated plasma protein (PAPP-A) -if just nuchal fold is incr., get 3D US and Fetal Echo in 2nd tri -if both abnormal, offer invasive testing: CVS or amnio. if either come back normal, still proceed w/ 2nd tri US & echo. if they deny invasive testing, in 2nd tri get 3D US & fetal echo. *2nd tri DS screening*: ONLY a serum blood test: either an *alpha feta protein (AFP)* OR a *quad screen (AFP + dimeric inhibin alpha)* -the AFP would be low in DS *(not 1st line)* -the quad would be elevated in DS *(1st line)* *ideal DS screening: both 1st & 2nd tri screening methods, unless you miss 1st tri then just do 2nd* -highest risk= age & positive results in 1st & 2nd tri; offered invasive testing (CVS or amnio) -mod risk= positive 1st tri, neg 2nd tri; undergo 2nd tri US -low risk= negative 1st tri; do not under 2nd tri screening *Cystic Fibrosis:* done as routine screening in EVERY mom d/t high # of mutations (MC is delta F508); MC in non-Hispanic White Americans and Ashkenazi Jews -->if either parent has CF, go straight to complete CFTR gene sequence

Diagnosing Pregnancy (not an objective)

often dx when women presents w/ at home positive pregnancy test which is then confirmed w/ a *urine or blood hCG (human chorionic gonadotropin*. US is often used, especially if concerned for miscarriage or ectopic. things to consider: -abrupt cessation of menstruation in a healthy reproductive woman who previously experienced spontaneous, cyclical, predictable menses is highly s/o pregnancy -b/c menstrual cycles vary, *amenorrhea is not reliable until at least 10 days after expected menses* -spotting may occur after conception which the pt may misinterpret as her cycle (still prompts workup if occurs in 1st trimester) *hCG=initial diagnostic test* -hCG is needed to support pregnancy by preventing involution of the CL, which is the main site of progesterone formation in the first 6 wks of pregnancy -consists of alpha & beta subunits -alpha unit: identical to LH, FSH, and TSH -*beta subunit is what's measured in pregnancy test* -can be detected in maternal serum or urine by *8-9 days after ovulation* vs home tests can only detect once menses missed -*doubling time is ~1.5-2 days*; level decreases & plateaus at ~16 wks since no longer needed to support pregnancy. *so after 16 wks, hCG is NOT a good indicator of how pregnancy is progressing* -other causes of elevation: molar pregnancy, cancer, exogenous hCG injxn (causes wt. loss), renal failure. -*if hCG has only incr. slightly but not doubled, consider ectopic* -*if hCG is increasing more than double, consider twins* -*if hCG is going up massively, consider molar pregnancy* *TVUS=confirmatory test* -*gestational sac= first US evidence of pregnancy.* it's a small eccentric (not-midline) fluid collection in the endometrial cavity (often look empty); seen on TVUS by ~4 wks gestation; try not to US before 4 wks since you won't see anything and mom freaks out -*pseudosac-* fluid collection in the endometrial cavity seen with *ectopic pregnancy*; seen in *midline* of endometrial cavity ; order hCG next -*yolk sac*-bright glowing echogenic ring w/ anechoic center seen ~5 wks; can see embryo w/ fluttering HR (160-180 bpm) -*embryo w/ fetal heart tones (FHT)*- see at 6 wks -*pregnancy of unknown location*- if US is ambiguous, do serial hCG levels to determine if normal, ectopic, or miscarriage -other signs of normal gestation sac= decimal sign or intradecidual sign: anechoic center surrounded by a single echogenic rim -up to 12 wks, the crown-rump length (CRL) is predicative of gestational age w/in 4 days

Given a mother with *gestational diabetes*, describe the expected maternal and fetal complications.

onset of glucose metabolism disorder during pregnancy -worse for baby than for mom *universal screening is most commonly done* -week 24-28: 50g OGTT (oral glucose tolerance test) . If abnormal, do 100g test 3 hours later. -if high risk (i.e. hx of gestational DM), can do 75g OGTT screen in wks 11-14 and then repeat it btwn wks 24-28 biggest concerns/consequences: -macrosomia causing shoulder dystocia or need for C/S -fetus hypoglycemia -maternal obesity -risk of DM remaining post delivery tx: -if FGS <95 or 2 hour values <120- first try diet -if above, offer insulin (0.7-1.0 units/kg/d). if refuse, oral meds (metformin or glyburide) are fine *gestational diabetics can do oral meds if they refuse insulin versus preexisting diabetics absolutely must use insulin* don't ask me why. workup: -*3rd tri: kick counts & biophysical profiles* count # kicks/10 mins (ensures baby isn't affected from high BS) -retest mom 6 wks post partum to ensure it has resolved

what would you use in a person who smokes, or has clotting history?

progestin only options

Syphilis in pregnancy

required by LAW at first prenatal visit, consider again in 3rd trimester and delivery(lots of false +'s)Tx: Pen G x 2 doses (even if mild PCN allergy)if suspicious lesion, do a c-section False positive exist in high numbers - work these up

*Gestation HTN* (not a specific objective)

this is HTN that develops *AFTER 20 weeks of gestation (20+0)* diagnostic criteria: -after 20 wks of gestation (20+0 counts) -*140/90 on 2 readings at least 10 mins apart* -*absence of proteinuria* mngmnt: -based on severity of reading (home health vs inpt based on evidence of preeclampsia) -the concern is the development of superimposed pre-eclampsia (syndrome of HTN AND evidence of organ damage, i.e. proteinuria) -consider hospitalization for initial workup -daily weights- retaining fluid? -24 hour urine & CrCl- proteinuria? -CMP- liver, kidneys -CBC -PT/PTT -fetal US w/ growth measurements and fluid levels complixns to fetus: placenta vessels under high pressure can cause intrauterine growth retardation, placental insufficiency, *oligohydramnios* (low amniotic fluid)

Describe the *RF, pathophys, clinical presentation & mngmnt for a patient with a placental abruption*.

this is when placenta separates from implantation sites (tear away of placental wall) *"her worst nightmare"* -can be partial/small -can be complete & massive--> LITERS of blood loss (mom & fetus can both die) -may not be evident until after delivery (placenta will have clot) *RF:* -trauma -drugs -HTN -smoking -PPROM(prolonged premature rupture of membranes) -fibroids: cause excess pressure on placenta -lupus anticoagulant -idiopathic *sx:* *sudden onset abdominal pain w/ MASSIVE bright red bleeding* *uterine tenderness* +/- hypovolemic shock (d/t massive blood loss) +/-DIC *dx:* fetal monitor 2 lines intersect= diagnostic ^if this is present, you have ~6 mins to fix this *mngmnt/tx:= delivery* -blood loss is massive, these pts. *can lose their ENTIRE Blood volume FAST* -DELIVERY: vaginal: if dead fetus and no massive hemorrhage C/S: compromised fetus or massive hemorrhage *sequelae:* -death of mom & fetus d/t loss of too much blood -severe neuro deficitis -cerebral palsy -DIC -sepsis

Describe the clinical presentation, US findings, & mngnmnt of *placental previa.*

this is when the placenta is implanted over or very near cervical os "placental migration" -if exists at 28 wks, likely to persist -->concerning because very big risk of hemorrhage sx: *painless bleeding at 28+ wks of gestation* dx: US shows low lying placenta *do NOT do pelvic/cervical exam, you can puncture the placenta* tx: 1. steroids- mature lungs 2. if near term, plan *C/S* (38 wks is usual plan) 3. if abnml placenta (i.e. accreta), deliver even if not near-term **these moms cannot do vaginal delivery, their "door" is blocked by placenta** **if previa is present, cannot have sex or insert anything**

Discuss screening for *HIV* in pregnancy

transmission= blood or bodily fluids -->delivery is MC time of transmission to fetus; breastfeeding is 2nd MC transmission route -->transmission is increased by presence of other dz's (i.e. HSV2) *biggest concern in pregnancy= viral load at time of delivery* -less than 400 copies= ideal -higher than 100K copies= bad *these moms should NOT breastfeed regardless of viral load at delivery* mgnmnt: *HAART Therapy:* all are category C but the benefits outweigh risk so TX PT! -limit the # of invasive procedures, cervical exams, forceps, operative delivery (?) -all routine labs -required: TB tests -non-life vaccines UTD (HBV, Tdap combo w/ pertussis) -CXR 1st tri, baby is by pelvis -liver fxn -Viral load delivery: -*if viral load >1K copies OR if no HAART use, schedule C/S at 38 wks* Try to deliver baby vaginally if viral load allows, this is better for baby

Placenta Previa, what is it and how do you treat it?

when the placenta is implanted over or very near the cervical os (partial previa)- great risk for hemorrhage PAINLESS BLEEDING- don't do a digital cervical exam if you suspect this management: - steriods: lung maturity- observe in hospital until no further bleeding; d/c on graduated activity- if near term plan scheduled c section (38 weeks), if not plan to deliver pre-term

Describe the contraindications for steroid contraceptives.

• *Estrogen- Progestin combo* o Pregnancy (or breastfeeding) o Uncontrolled hypertension oSmokers older than 35 years o Diabetes with vascular involvement o Cerebrovascular or coronary artery disease o Migraines with associated focal neurologic deficits o Thrombophlebitis or thromboembolic disorders o History of deep-vein thrombophlebitis or thrombotic disorders o Thrombogenic heart arrhythmias or thrombogenic cardiac valvulopathies o Undiagnosed abnormal genital bleeding o Known or suspected breast carcinoma o Cholestatic jaundice of pregnancy or jaundice with pill use o Hepatic adenomas or carcinomas or active liver disease with abnormal liver function o Eondometrial cancer or other known or suspected estrogen-dependent neoplasia *This will be important on the test, if the patient is a smoker over 35, cardo hx, clotting hx, hepatic hx, they can't take combo methods* •*Progestin only* o Active breast cancer o Severe decompensated cirrhosis o Malignant liver tumor o Past breast cancer o Systemic lupus erythematosus with antiphospholipid antibodies

List the alternative uses for each class of steroid contraception.

• *Estrogen- Progestin combo* o Reduction in dysmenorrhea, menorrhagia o Reduction in pelvic pain associated with endometriosis o Reduction in risk of ectopic pregnancy o Reduction in symptoms of PMS, PMDD o Reduction in risk of benign breast disease, new ovarian cysts o Reduction in risk of ovarian cancer, endometrial cancer, colorectal cancer o Reduction in moderate acne, hirsutism • *Progestin only* None

Compose a counselling plan regarding the use of steroid contraceptives.

• *Estrogen- Progestin combo* aka COC (combination oral contraception) o May lead to potassium retention (hyperK), so you monitor levels at 1 month. o More helpful with PMDD (premenstrual dysphoric disorder) o More helpful with acne *Administration of COC:* - (first choice) Start on first day of menstrual cycle (Because then additional contraception not needed) - (second choice) Start on first Sunday after period, or as soon as you get the pills (need 7 days back up protection) *What to do if you miss a dose of COC?:* - 1 pill missed, take 2 the next day to minimize breakthrough bleeding. (conception still unlikely no back up needed) - several doses missed, take 2 pills when noticed, backup method needed for 7 days - COC are not tetatogenic, so don't fret *Skipping placebo week*: - You can not take the nuva ring out or leave the patch on instead of removing for 1 "placebo week" - Some pill packs now come with 28 hormone pills and no placebo, others you can ignore the placebo pills and start the next pack - If you start to get breakthrough bleeding, take the nexk placebo week, then start all over again. - overall, fewer menstrual symptoms - May help fix menstrual related anemia - Some women get too paranoid about pregnancy scares and they don't like this. • *Progestin only* o The Pills: "Mini-pill" -Taken daily without pill-free interval (no placebo weeks) -Must take at the same time daily -Need back up contraception x 48 hrs if greater than 3 hr delay (because effects only last 24 hours, best at 22 hours, so take in the morning) - Higher chance than combo of breakthrough bleeding *Progestin only for breastfeeders, cardio, and vascular history (exception is hx of acute DVT/PE should only get copper IUD)*

Describe the classes of steroid contraception.

• Estrogen- Progestin combo • Progestin only No estrogen only options

Discuss the mechanism of action for each class of steroid contraception.

•*Estrogen- Progestin combo* o BOTH: Inhibit midcycle surge of gonadotropin secretion o Progestin: Supress ovulation by suppressing LH o Progestin: Alter endometrial receptivity (decrease it) o Progestin: Inhibit ability of sperm to access the upper genital tract (by changing cervical mucous) o Estrogen: Suppress FSH and stabilize endometrium to prevent intermenstrual bleeding. o *EXAMPLES:* Pills, transdermal patch, vaginal ring With combo pills, the progestin can stay constant or fluctuate. o Constant = monophasic pills o Fluctuating = multiphasic (biphasic and triphasic pills) the progestin starts low and gets higher, more breakthrough bleeding o If the pill name ends in Fe, the placebos have iron in them. • *Progestin only* o Thickening of cervical mucous , suppress LH, render endometrium unfavorable o Don't always prevent ovulation, since that is Estrogen o *EXAMPLES:* Pill, injectable, implants


Conjuntos de estudio relacionados

Vocabulary Words ~Let Wild Animals be Wild and Don't Release Animals Back to the Wild

View Set

Life & Health Insurance License Test

View Set

Zoheed 10a - Sexual Reproduction and meiosis

View Set

AP Environmental Science MCQ Unit 1 Test - Mrs. DiCrisi

View Set